Simulacro 1

! 1. EL TEST DE LA D-XILOSA ALTERADO INDICA: 1. DEFECTO DE LA DIGESTIÓN DE LOS CARBOHIDRATOS. SÍNDROME DE MALABSORCIÓN

Views 106 Downloads 0 File size 9MB

Report DMCA / Copyright

DOWNLOAD FILE

Recommend stories

Citation preview

!

1. EL TEST DE LA D-XILOSA ALTERADO INDICA: 1. DEFECTO DE LA DIGESTIÓN DE LOS CARBOHIDRATOS. SÍNDROME DE MALABSORCIÓN POR DEFECTO DE LA PARED DEL 2. 2. INTESTINO. 3. DÉFICIT CONGÉNITO DE DISACARIDASAS. 3. 4. OBSTRUCCIÓN BILIAR. 4. Gráfico de respuestas 1.

Comentario La D-xilosa se absorbe en el yeyuno por difusión pasiva, sin necesidad de que intervengan las enzimas pancreáticas o las sales biliares. Por ello, está alterada en procesos que afectan a la pared del tubo digestivo (respuesta 2 correcta), sin verse influida por alteraciones pancreáticas ni biliares. El test de la D-xilosa consiste en administrar este azúcar en ayunas, y luego se mide su presencia en orina, cuando pasan unas horas. Si la encontramos en orina, quiere decir que se ha absorbido a nivel intestinal.(R/2)

! 2. Un marinero de 37 años acude a consulta por presentar dos úlceras de aproximadamente l cm de diámetro en el orificio del prepucio. Son dolorosas, de bordes irregulares y el fondo de las úlceras está cubierto de exudado necrótico. En la exploración física aparece una adenopatía inguinal izquierda inflamatoria de carácter doloroso y sensible al contacto. El resto de la exploración es normal. El paciente reconoce haber mantenido contactos sexuales con prostitutas cada vez que su barco fondeaba en un puerto, siendo la última vez hace una semana. ¿Cuál sería su diagnóstico? 1. Chancro blando. 2. Linfogranuloma venéreo. 3. Sífilis primaria. 4. Herpes genital. Gráfico de respuestas

1. 2. 3. 4.

Comentario Dentro de las ETS, es importante que sepa realizar el diagnóstico diferencial entre ellas. Veamos opción por opción: Chancro blando: Produce lesiones múltiples, dolorosas y con adenopatías unilaterales. Sífilis primaria: El chancro suele ser único, no doloroso, y con adenopatías bilaterales. Linfogranuloma venéreo: No produce un chancro como tal, sino una lesión papulosa, o incluso nada en absoluto… Pero después aparecen adenopatías unilaterales de gran tamaño, que acaban fistulizando. Herpes genital: Lesiones ulcerosas múltiples, con adenopatías bilaterales y molestias dolorosas.

! !

! En este caso, la presencia de una adenopatía unilateral, así como el dolor y el período de incubación breve (una semana) habla a favor de un chancro blando.(R/1)

! 3. Mujer de 45 años que tras intervención quirúrgica de masa ovárica izquierda presenta dolor en flanco izquierdo, náuseas, vómitos, ileo paralítico e hidronefrosis izquierda. ¿La prueba que usted pediría para valorar la lesión que probablemente tenga la paciente, sería? 1. USG abdominal. 2. USG transvaginal. 3. Arteriografía. 4. Urografía. Gráfico de respuestas

1. 2. 3. 4.

Comentario Es una pregunta de dificultad media que debe ser tenida en cuenta. Se trata de un posible ligadura de uréter que ocasiona hidronefrosis con fiebre, dolor en flanco, náuseas, vómitos e íleo. Se podría estudiar mediante USG, pero el mejor método diagnóstico en estos casos es la urografía ya sea intravenosa o retrógrada. En cuanto al tratamiento, la primera mediad debe se la derivación urinaria.(R/4)

4. Excluyendo la válvula aórtica bicúspide, ¿cuál es la cardiopatía congénita más frecuente?: 1. Conducto arterioso persistente. 2. Comunicación interventricular. 3. Coartación aórtica. 4. Tetralogía de Fallot. Gráfico de respuestas

1. 2. 3. 4.

Comentario Es importante que domine bien su clasificación así como los aspectos clínicos de las principales cardiopatías congénitas. Recuerde que la cardiopatía congénita más frecuente es la válvula aórtica bicúspide, seguida de la CIV. Ahora bien, si le preguntan por la cardiopatía congénita cianótica más frecuente debe contestar la transposición de los grandes vasos, en el RN y la tetralogía de Fallot a partir del primer año de vida.(R/2)

5. Una de las siguientes NO es causa habitual de hemoptisis: 1. Bronquiectasias. 2. Tuberculosis. 3. Estenosis mitral. 4. Enfisema pulmonar. Gráfico de respuestas

1. 2. 3. 4.

Comentario El conocimiento de la semiología en medicina es muy importante porque es necesario dominar determinados síntomas para así conseguir seleccionar la respuesta correcta en un caso clínico, ya sea porque lo conocemos o simplemente descartando el resto de las opciones. La hemoptisis se

! !

! define como la presencia de sangre en el esputo, normalmente procedente de vías aéreas subglóticas, y siempre es motivo de preocupación tanto para el enfermo como para el médico. Esta sangre puede proceder de las vías respiratorias, del parénquima pulmonar o de los vasos sanguíneos. Las enfermedades del árbol respiratorio pueden ser inflamatorias (bronquitis, que es la causa más frecuente de hemoptisis, bronquiectasias, fibrosis quística,...) o neoplásicas (carcinoma broncógeno o carcinoide). Las enfermedades del parénquima pueden ser localizadas (neumonía, absceso, tuberculosis o Aspergillus) o difusas (sd de Goodspasture y hemosiderosis pulmonar idiopática). Las enfermedades vasculares que pueden cursar con hemoptisis son el TEP, las malformaciones y todas aquellas situaciones que aumenten la presión en el árbol pulmonar (p.e. estenosis mitral). De esta forma vemos que el enfisema no suele producir hemoptisis, por lo que la seleccionamos como opción correcta.(R/4)

6. De los siguientes, el factor más relacionado con el crecimiento intrauterino retardado tipo simétrico es: 1. Infección congénita. 2. Hipertensión. 3. Diabetes con vasculopatía. 4. Edad de la mujer por encima de los 35 años. Gráfico de respuestas

1. 2. 3. 4.

Comentario Los fetos pequeños para la edad gestacional son aquellos que se encuentran debajo del percentil 10. Se dividen en:

PEQUEÑO PARA EDAD GESTACIONAL CONSTITUCIONAL: no hay alteraciones estructurales, son normales pero en rango de crecimiento bajo.

PEQUEÑO PARA LA EDAD GESTACIONAL ANOMALO (CIR SIMETRICO): Presentan anomalias genéticas estructurales o secundarias a una infección TORCH. Son "armónicos". Todos los parámetros son proporcionados pero retrasados respecto a la edad gestacional.

CIR II O ASIMÉTRICO: Secundarios a una funcion placentaria alterada. Doppler alterado "les falta comida". Son disarmónicos con BDP y LF acorde con la edad pero DA retardado.(R/1)

7. Lactante de 2 meses y medio, que presenta un cuadro de catarro de vías altas de 2 días de evolución, acude hoy al servicio de urgencias por rechazo de las tomas y dificultad respiratoria. En la inspección tiene tiraje subcostal e intercostal moderado, con ruidos inspiratorios de vías altas; se auscultan roncus y sibilancias diseminadas en ambos hemitórax; el resto de la exploración es normal. Tiene una saturación de O2 del 90%. ¿Qué patología sospecha? 1. 2. 3. 4.

! !

1. 2. 3. 4.

Bronquiolitis aguda. Bronquiolitis obliterante. Cardiopatía congénita descompensada. Asma bronquial.

! Gráfico de respuestas

Comentario Pregunta muy fácil y que no debe fallar, por lo típico y lo frecuente del cuadro clínico. Aunque la definición de bronquiolitis habla de lactantes menores de dos años con su primer episodio de dificultad respiratoria, se da más en los menores de 6 meses. Es importante tener en cuenta los síntomas previos para su diagnóstico, porque en los lactantes más mayores, sobre todo con antecedentes familiares u otros factores de riesgo para hiperreactividad bronquial, como la dermatitis atópica, las alergias alimentarias, o el tabaquismo pasivo, la bronquiolitis puede confundirse con el asma. Las crisis de broncoespasmo, sin embargo, suelen tener una aparición brusca; en la auscultación, se escuchan fundamentalmente sibilancias (sin ruidos de secreciones), y lo más importante, las crisis se repiten con frecuencia.(R/1)

! 8. En una postmenopáusica sana, el tratamiento sustitutivo adecuado es: Antidopaminérgicos tipo veralipride para el síndrome climatérico y asociar alendronato. 2. Fitoestrógenos. 2. 3. Estrógenos y progestágenos. 3. 4. Asociación de fitoestrógenos con bifosfonatos y calcio con vitamina D3. 4. Gráfico de respuestas 1.

1.

Comentario Pregunta sobre el tratamiento de la menopausia que puede resultar algo complicada si no tenemos los conceptos claros. No hay que confundirse por la variedad de respuestas, ya que en el enunciado tenemos la clave: postmenopáusica sana. En las opciones 1 y 4 se incluyen tratamientos para la osteoporosis (raloxifeno y bifosfonatos como el alendronato), con lo que ya no se trata de mujeres sanas. Los fitoestrógenos son sustancias de origen vegetal con acción estrogénica débil, con lo que será efectivas en un grupo reducido de pacientes que presenten síndrome climatérico débil. Es obvio pensar que el tratamiento sustitutivo adecuado es la combinación de estrógenos y progestágeno, ya que mejora los síntomas del síndrome climatérico y es útil en la prevención de la pérdida de masa ósea; así, la opción correcta es la nº4.(R/3)

! 9. Señale cuál de los siguientes síntomas NO aparecería en una fractura-luxación del D12 con sección medular, de muy poco tiempo de evolución: 1. Hiperreflexia. 2. Atonía muscular. 3. Pérdida completa de la sensibilidad por debajo de nivel lesionado. 4. Abolición de reflejos superficiales. Gráfico de respuestas

1. 2. 3. 4.

Comentario Pregunta difícil y de un tema poco preguntado como es el manejo de los traumatismos medulares. La sección medular de poco tiempo de evolución se corresponde con el denominado choque medular, que consiste en la pérdida transitoria de todos los reflejos espinales con parálisis fláccida por debajo del nivel de lesión (opción 1 incorrecta). Recuerde que el choque medular precede en

! !

! un tiempo variable a la lesión medular establecida, caracterizada por signos de 1ª motoneurona por debajo de la lesión (espasticidad e hiperreflexia).(R/1)

10. Hombre de 62 años que consulta por molestias faríngeas de 3 meses de evolución. A la exploración se aprecia tumoración ulcero-vegetante de cara laríngea de epiglotis que infiltra bandas ventriculares. Cuerdas vocales libres con movilidad conservada. Adenopatías bilaterales múltiples, pequeñas, móviles y no dolorosas. El diagnóstico anatomopatológico es carcinoma epidermoide moderadamente diferenciado. ¿Cuál de las siguientes opciones terapéuticas considera más CORRECTA?: 1. Laringuectomía horizontal supraglótica y vaciamiento radical bilateral. 2. Hemilaringuectomía vertical y vaciamiento funcional bilateral. 3. Laringuectomía horizontal supraglótica y vaciamiento funcional bilateral. 4. Hemilaringuectomía vertical y vaciamiento radical bilateral. Gráfico de respuestas

1. 2. 3. 4.

Comentario Respecto al tratamiento de los tumores supraglóticos, la cirugía es uno de los tratamientos más utilizados. El tumor descrito es un tumor supraglótico bajo (supraglotico infrahioideo) que respeta el plano glotico a nivel de comisura anterior, cuerda vocal y aritenoides y además no paraliza la cuerda vocal, con lo cual la cirugía a realizar será una laringuectomia horizontal supraglotica. El estado del cuello es el factor que empeora más el pronostico en estos tumores, ya que las metástasis regionales cervicales son frecuentes y hace necesario un vaciamiento cervical bilateral en este paciente que es N2, pero también estaría indicado aunque fuese N0. El vaciamiento si sólo es un N2 y no afecta a paquete vascular cervical suele ser suficiente con que sea funcional.(R/3)

11. El CO2 es el gas ideal para la realización de un neumoperitoneo en una laparoscopia, ya que para que un gas sea utilizado en laparoscopia debe ser: 1. Medianamente soluble, irritante e incomburente. 2. Insoluble, comburente e incoloro. 3. Soluble, incoloro e incomburente. 4. Incoloro, insoluble y volátil. Gráfico de respuestas

1. 2. 3. 4.

Comentario Se han empleado para el neumoperitoneo diferentes tipos de gases, desde aire, nitrógeno, argón, helio, CO2 y óxido nitroso. Cada uno de ellos reviste características distintas, pero en términos generales sólo los gases solubles (CO2 y N2O) tienen aplicaciones en la práctica de la laparoscopia. A diferencia del N2O, el CO2 es incombustible, evitando el riesgo de chispazos al usar el electrobisturí.(R/3)

12. Una mujer de 85 años consulta por cansancio y debilidad especialmente por las mañanas. A veces se encuentra inestable al caminar y tiene que sentarse para recuperar el equilibrio. En dos ocasiones ha tenido que sentarse para no caer pero niega síntomas de mareo. Tiene hipertensión arterial, incontinencia urinaria y artrosis. Su tratamiento es hidroclorotiazida (25 mg/d), oxibutinina (10 mg/d), lisinopril (10 mg/d), calcio (1500 mg/d) y paracetamol (3 g/d). A la exploración destaca una tensión arterial de 115/70 mmHg, pulso 80 lpm. Sus movimientos son lentos. Tiene un temblor moderado en las manos. Puede levantarse de la silla lentamente pero sin necesidad de apoyarse en los brazos. Camina levemente inclinada hacia delante con poco balanceo de los brazos. Gira

! !

! lentamente pero sin perder el equilibrio. No es capaz de mantenerse sobre un solo pie. ¿Cuál de las siguientes posibles actuaciones realizaría en primer lugar? 1. Valorar la agudeza visual. 2. Estudio con mesa basculante. 3. Medir la tensión arterial tumbada y levantada. 4. Intento terapéutico con L-dopa. Gráfico de respuestas

1. 2. 3. 4.

Comentario Pregunta difícil. El caso clínico nos describe a una mujer de 85 años con clínica probablemente debida a una enfermedad de Parkinson (cansancio, debilidad, inestabilidad con la marcha, bradicinesia, temblor distal, escaso balanceo con la marcha…). Sin embargo, la paciente consulta por cansancio y debilidad que aparecen “especialmente por las mañanas”, y además “ha tenido que sentarse para recuperar el equilibrio y para no caer” negando síntomas de mareo. Estos datos nos hacen sospechar un cuadro de hipotensión postural en el contexto de una posible enfermedad de Parkinson. Además, la presencia de tratamiento antihipertensivo, especialmente los diuréticos, hablan a favor de esta sospecha. También toma un anticolinérgico, que puede estar contribuyendo (oxibutinina). La clave de esta pregunta, viene dada por el enunciado final: nos pide la actuación en primer lugar. Las respuestas 1 y 2 son claramente falsas. La disminución de la agudeza visual no produce este cuadro, no existen datos de focalidad neurológica que nos hagan sospechar que una RM aportará información al cuadro, y la mesa basculante es un test realizado para el diagnóstico diferencial del síncope vasovagal con riesgo de hipotensión grave y asistolia que no debe realizarse en ancianos. La respuesta 4 es también falsa, aunque lleva a confusión. Es cierto que al tratarse de una anciana con clínica parkinsoniana el intento con L-dopa es una buena actitud a llevar a cabo. Sin embargo, en la pregunta se nos pide la primera actuación, y en el contexto de una posible hipotensión postural, es prioritario descartar esta y tratarla, disminuyendo por ejemplo la dosis de diurético. Además, el tratamiento antiparkinsoniano con L-dopa puede empeorar o causar hipotensión ortostática, haciéndose menos recomendable en este cuadro. Por tanto, la respuesta correcta es la 3, medir la tensión arterial tumbada y levantada, siendo este el test más fácil y rápido para confirmar el ortostatismo.(R/3)

13. A 4-year-old boy with a history of sickle cell anemia and splenic infarcts becomes to the emergency department presenting with fever and malaise. Laboratory results shows anemia, reticulocytosis and leukocytosis. What would be expected in a peripheral blood smear? 1. Howell Jolly Bodies. 2. Heinz bodies. 3. Dacryocites. 4. Hipersegmented PMN. Gráfico de respuestas

1. 2. 3. 4.

Comentario ! !

! Howell-Jolly bodies. Howell-Jolly bodies are histopathological findings of basophilic nuclear remnants (clusters of DNA) in circulating red blood cells.They usually indicate spleen damage. In patients affected by sickle cell anemia hyperviscosity causes spleen infarcts so its function gets markedly reduced.(R/1)

14. ¿Cuál de los siguientes NO es un factor asociado a candidiasis? 1. Hemorragia intermenstrual. 2. Diabetes mellitus. 3. Corticoterapia. 4. Antibióticos. Gráfico de respuestas

1.

2. 3. 4.

Comentario Si se fija la respuesta 2 o 3 puden causar inmunosupresión celular, por lo que en estos pacientes se incrementa el riesgo. El uso de antibióticos aumenta las probabilidades de candidiasis por destruir la flora bacteriana habitual. La respuesta que debe elegir es la 1.(R/1)

15. Con la clasificación de choque hipovolémico, por ATLS, señale verdadero o falso: ( 1 ) Pérdidas de sangre Clase I, son equivalentes a la donación sanguínea. ( 2 ) En Clase II, hay marcado compromiso como taquicardia, taquipnea e hipotensión arterial. ( 3) Pérdidas Clase III, pueden ser manejadas solo con solución salina. ( 4 ) Pérdidas Clase IV, se pierde aproximadamente 40% de la volemia y ponen en riesgo la vida del paciente. 1. FFVV. 2. VFFV. 3. FVFV. 4. VFVF. Gráfico de respuestas

1. 2. 3. 4.

Comentario La opción correcta es la 2 ya que: -Clase I: V. Las pérdidas son equivalentes a la donación al ser menores de 750ml. -Clase II: F. No hay marcado compromiso cardiovascular ya que la TA es normal y la taquicardia no es muy marcada. -Clase III: F. Se deben administrar cristaloides y concentrados de hamatiesen este nivel. -Clase IV: V. Las perdidas son del 40% del volumen o mayores con riesgo vital(R/2)

! !

! 16. Médico traumatólogo de 40 años que en una serología rutinaria se le detecta niveles de antiHBs de 30 UI/I. Refiere vacunación completa según pauta estándar frente hepatitis B hace 4 años. Sería recomendable: 1. Reiniciar pauta vacunal (0-1-6 meses). 2. Reiniciar pauta vacunal (0-1-2-12 meses). 3. No revacunar. Darle inmunoglobulina anti-hepatitis B y reiniciar pauta vacunal (0-1-2-12 4. 4. meses). Gráfico de respuestas 1. 2. 3.

Comentario El paciente presenta niveles protectores de AcHBs (>10UI/l) por lo que no requiere más inmunización frente a la hepatitis B.(R/3)

17. A 43-year-old woman with a history of multinodular goiter comes to the clinic for a follow-up examination after a gammagraphy has been performed, which is shown in the image. Laboratory studies show: TG and TPO antibodies slightly elevated, TSH level 0.001 microUi/ml (normal range: 0.35-4.95), free T4 3.569 ng/dl (normal 0.700-1.480) and free T3 > 30 pg/ml (normal 1.71 4.53). Which of the following is the most likely diagnosis? 1. Graves disease. Functioning metastatic follicular carcinoma of the thyroid gland. Thyroglobulin 2. 2. levels should be measured. 3. 3. Toxic multinodular goiter. 4. 4. Toxic thyroid adenoma. Gráfico de respuestas 1.

Comentario Para acertar esta pregunta, dependemos bastante de la imagen gammagráfica adjunta. En cualquier caso, no resulta difícil darse cuenta de que se trata de un adenoma tóxico (tiroides prácticamente abolido, con una enorme hipercaptación en una zona redondeada). Este diagnóstico encaja perfectamente con el perfil hormonal que nos describen (elevación de las hormonas tiroideas con TSH suprimida, es decir, el de un hipertiroidismo). Si se tratara de un bocio multinodular tóxico, la captación se vería en forma de múltiples nódulos, como su nombre indica.

! !

! Tampoco podría tratarse de un estruma ovari, donde tendríamos una gammagrafía hipocaptante (es un tumor ovárico que secreta hormonas tiroideas y por ello la TSH estaría suprimida). Es difícil pensar en una metástasis de un cáncer folicular, ya que estamos a nivel del tiroides, donde en todo caso sería un tumor primario… Y, aun así, sería muy difícil pensar en un cáncer, por tratarse de un nódulo “caliente”. Por último, la enfermedad de Graves-Basedow produciría una hipercaptación tiroidea, pero difusa.(R/4)

18. En un paciente que presenta una eritrodermia, los siguientes signos clínicos harían sospechar una etiología psoriásica EXCEPTO uno, señálelo: 1. Límite neto de las lesiones. 2. Exfoliación seca de todo el tegumento, incluido el cuero cabelludo. 3. Estado general conservado. 4. Prurito marcado en las localizaciones que la psoriasis afecta típicamente. Gráfico de respuestas

1. 2. 3. 4.

Comentario Se trata de un pregunta complicada, que exige un conocimiento bastante concreto de las características de un tipo de psoriasis no muy preguntada en el ENARM. La eritrodermia psoriásica no es uno de los apartados más preguntados. Debe recordar al menos: - definición de eritrodermia: enrojecimiento generalizado, con descamación que no representa una entidad en sí, si no que refleja una presentación clínica de diferentes entidades. - En este caso nos preguntan por la causa psoríasica de la misma. La psoriasis representa en 20% del total de las causas de eritrodermia. Suele precederse de lesiones típicas de psoriasis en placas y suele ser también el resultado del cese de tratamientos como el metotrexate o corticoides sistémicos. - Clínicamente puede orientar a su diagnóstico: o cambios unguelaes o descamación plateada en placas delimitadas en cara, rodillas y codos no de forma generalizada.(R/4)

19. ¿Cuál de las siguientes situaciones NO predispone al Kernicterus en el recién nacido?: 1. Ingesta materna de sulfamidas. 2. Sepsis neonatal. 3. Hipoalbuminemia neonatal. 4. Alcalosis metabólica. Gráfico de respuestas

1. 2. 3. 4.

Comentario El kernicterus es la consecuencia del depósito de bilirrubina indirecta sobre los ganglios basales del SNC. Entre sus causas se necesita una situación de hiperbilirrubinemia muy importante (debida a hemólisis, sepsis, hemorragia interna, TORCH) y factores adicionales (como prematuridad, acidosis metabólica, hipoxia, enfermedad de base, ingesta materna de sulfamidas, hipoalbuminemia,...)(R/4)

20. De las siguientes características, una de ellas NO es característica de la intolerancia a la lactosa:

! !

! 1. Lactato fecal aumentado. 2. Heces acuosas y con olor ácido. 3. Deposiciones explosivas. 4. Cuerpos reductores (Clinitest) negativos en heces. Gráfico de respuestas

1. 2. 3. 4.

Comentario La intolerancia transitoria a la lactosa es un cuadro autolimitado en el tiempo que aparece tras una gastroenteritis invasiva , debido a la destrucción de los enterocitos. La incapacidad de digerir la lactosa hace que este azúcar permanezca en heces. Al ser la lactosa un azúcar reductor, se acidifican las heces, apareciendo diarrea explosiva y eritema anal por irritación, aumentando el lactato fecal y apareciendo en el Clinitest presencia mayor del 0.5% de cuerpos reductores, es decir, test POSITIVO, por eso la respuesta 4 es la única falsa.(R/4)

21. Señale en qué supuesto hay que estudiar siempre la causa de la ictericia: 1. La ictericia que aparece a las 48 horas de vida. 2. La bilirrubina alcanza una cifra superior a 10 mg/dL. 3. La bilirrubina directa supera el 20% de la total en cualquier momento. 4. En el prematuro alcanza un valor superior a 12 mg/dL al 7º día de vida. Gráfico de respuestas

1. 2. 3. 4.

Comentario Esta pregunta puede parecer tediosa y compleja, pero no lo es tanto si tiene las nociones claras. Ha de saber que las hiperbilirrubinemias pueden ser de dos tipos en función de la cifra de bilirrubina directa: colestáticas (si BID excede el 20%) y no colestáticas (si BID menor del 20%). La opción 3 designa una ictericia colestática, tributaria por tanto de estudio. El resto de las opciones forma parte del espectro de la ictericia fisiológica.(R/3)

22. La edad más frecuente de convulsión febril es: 1. 1 mes - 4 años. 2. 6 años - 10 años. 3. 3 meses - 5 años. 4. 7 dias - 5 años. Gráfico de respuestas

1. 2. 3. 4.

Comentario Es simplemente un dato memorístico, o la sabe o no. La respuesta correcta es la número 3: 3 meses a 5 años.(R/3)

23. Un RN perderá calor por radiación si: 1. 2. 3. 4.

1. 2. 3. 4.

Esta expuesto al aire. No se le seca correctamente. Sus ropas están muy frías. Se le desnuda en un ambiente frio.

Comentario La reanimación neonatal es un tema que debe dominar.

! !

! Pregunta sencilla. La respuesta correcta es la 4. Repase la siguiente lista. Evaporación: a través de piel y pulmones. Conducción: hacia sólidos que están en contacto. Radiación: hacia sólidos que no hacen contacto. Convección: diferencia de temperatura con el aire circundante.(R/4)

24. Marque V ó F, respecto a la meningoencefalitis bacteriana: ( ) La glucosa aumenta en el LCR, ( ) Las proteínas aumentan en el LCR, ( ) Los gérmenes ingresan al encéfalo por los plexos coroideos, ( ) Para disminuir el edema cerebral usamos los corticoides, ( ) La ceguera como complicación es muy frecuente. 1. F V V F F. 2. F V F V V. 3. V V V V F. 4. F F F F F. Gráfico de respuestas

1. 2. 3. 4.

Comentario La meningitis bacteriana se caracteriza por: - Disminución de la glucosa en el LCR. - Proteinas aumentadas en el LCR. - Los gérmenes entran por los plexos coroideos. - No se usan corticoides para reducir el edema cerebral - No es frecuente la ceguera como complicación sino la sordera.(R/1)

25. Un paciente alcohólico, vagabundo, acude a Urgencias por síndrome febril de larga evolución y deterioro del estado general. A la exploración el paciente está sucio y descuidado y presenta un soplo sistólico en foco aórtico. Se realiza un ecocardiograma que demuestra la presencia de una vegetación en la válvula aórtica, a las dos semanas los hemocultivos continúan negativos. Este paciente presenta una endocarditis por: 1. Bartonella quintana. 2. Streptococcus bovis. 3. C. albicans. 4. S. aureus. Gráfico de respuestas

1. 2. 3. 4.

Comentario

! !

! La endocarditis infecciosa (EI) es un tema muy importante en el examen. Lo más preguntado es la etiología, clínica, tratamiento y profilaxis. Nos describen una EI en la que los hemocultivos son negativos, y de las 4 respuestas la más probable es la Bartonella quintana (RC- 1), que se ha descrito con mayor frecuencia en alcohólicos indigentes, cono el paciente de este caso. El cuadro se conoce como “fiebre de las trincheras”. Otros microorganismos que producen endocarditis infecciosa con hemocultivos negativos son los bacilos del grupo HACEK, que precisan medios de cultivo enriquecidos e incubación durante un tiempo prolongado. Coxiella y Brucella son también gérmenes productores de EI con hemocultivos negativos, suelen afectar a la válvula aórtica y, con frecuencia, precisan cirugía para su tratamiento.(R/1)

26. Niño de 3 años, previamente sano, en el cual al examen físico presenta fiebre, cefalea, signos meningeos y fotofobia, cual sería la etiología más probable: 1. Glaucoma agudo. 2. Neuritis óptica. 3. Meningitis. 4. Rabia. Gráfico de respuestas

1. 2. 3. 4.

Comentario No puede equivocarse en esta pregunta. La respuesta correcta es la 3 meningitis por presentar fiebre, cefalea, fotofobia y signos meningeos.(R/3)

27. ¿Cuál de las siguientes opciones caracteriza la fase aguda de las dermatitis eccematosas?: 1. Eritema, edema y vesículas. 2. Formación de costras y descamación. 3. Aparición de estrías de Wickham. Pápulas y placas eritematosas bien delimitadas y cubiertas por varias capas de 4. 4. escamas finas y nacaradas. Gráfico de respuestas 1. 2. 3.

Comentario El eccema es una reacción cutánea inflamatoria que se caracteriza histológicamente por espongiosis, acantosis más o menos marcada y un infiltrado linfohistiocitario perivascular superficial. La espongiosis es el edema intercelular de la epidermis que conduce a la ruptura de las uniones intercelulares con la formación de vesículas. Los eccemas pueden dividirse, desde el punto de vista evolutivo, en tres fases. En la fase aguda predomina la espongiosis y la formación de vesículas. Clínicamente esto se traduce en la presencia de eritema, edema, vesículas, ampollas y exudación. En la fase subaguda la espongiosis y las vesículas disminuyen mientras aumenta la acantosis. Además se forma un estrato córneo paraqueratósico. En la clínica se observan lesiones costrosas y descamación. Finalmente en la fase crónica se encuentra hiperqueratosis con áreas de

! !

! paraqueratosis y acantosis que clínicamente se manifiesta como grietas, fisuras y liquenificación. En el liquen plano se forma un reticulado blanquecino en la superficie de las pápulas que se denomina "estrías de Wickham". La opción 4 corresponde a las lesiones de la psoriasis.(R/1)

28. Una mujer de 55 años acude a consulta por presentar rectorragia con heces de aspecto mucoide. En el examen endoscópico se encuentra una tumoración plana de superficie vellosa, de 4 x 3 cm, que se reseca endoscópicamente aparentemente de forma completa. ¿Cuál es la actuación médica prioritaria en la paciente? Hacer un estadiaje tumoral inmediato, sobre todo con el fin de descartar metástasis hepáticas precoces. Realizar como tratamiento inmediato resección intestinal de la zona que incluya 2. el lecho tumoral, precedida de quimioterapia neoadyuvante y radioterapia 2. preoperatoria. Realizar un estudio histológico exhaustivo de la pieza con el fin de descartar áreas 3. 3. de carcinoma intramucoso o infiltrante. Realizar un estudio clínico, radiológico y genético urgente, con el fin de descartar 4. 4. poliposis múltiple familiar y cáncer colorrectal hereditario. Gráfico de respuestas 1.

1.

Comentario Estamos ante una mujer de edad media con rectorragia y el hallazgo de una tumoración que se reseca. Ante todo, la actitud prioritaria será filiar el tipo de lesión con el que nos encontramos, ya que de ello dependerá nuestra actitud posterior (opción 3 correcta). Lo primordial a descartar es la existencia de focos de malignidad, que no puede presuponer de antemano (opciones 1 y 2 incorrectas). Por último, en general las poliposis familiares no debutan en este tipo de edades y hay otros datos clínicos añadidos para ponernos tras la pista (opción 4 incorrecta).(R/3)

! 29. En una mujer embarazada de 18 semanas, sin antecedentes médicos de interés, con correcto seguimiento de la gestación y sin patología de interés durante las primeras 18 semanas, con niveles de beta-HCG dentro de la normalidad, PAPP-A dentro de la normalidad, glucoproteína b-1 específica de la gestación en valores de normalidad, detectamos una alfafetoproteína sérica elevada. ¿Cuál es la malformación congénita que más probablemente detectemos en el feto? 1. Atresia esofágica. 2. Onfalocele. 3. Mielomeningocele. 4. Gastrosquisis. Gráfico de respuestas

1. 2. 3. 4.

Comentario La elevación de la alfafetoproteína se relaciona estrechamente con las alteraciones del tubo neural, como el mielomeningocele (respuesta 3 correcta). El resto de las opciones también producirían elevación de esta proteína, pero si nos preguntan la malformación congénita más probable, debemos marcar las de carácter neurológico.(R/3)

! !

! 30. Sobre la atención inmediata del RN, marque la CORRECTA: 1. El score APGAR determina la conducta a seguir en el RN. 2. La aspiración traqueal siempre es mandataria y obligada. 3. La dosis de vitamina K es: 0.1 mg intramuscular. 4. El APGAR expresa la buena adaptación vital del RN a la vida extrauterina. Gráfico de respuestas

1. 2. 3. 4.

Comentario Respuesta 1 falsa. El Apgar no determina la conducta a seguir en el RN. Respuesta 2 falsa. No siempre habrá que realizar aspiración traqueal. Tiene indicaciones precisas. Respuesta 3 falsa. La dosis de vitamina K es de 0.5 a 1 mg IM.(R/4)

31. ¿Cuál de los siguientes NO es factor de riesgo de osteoporosis?: 1. Obesidad. 2. Multiparidad. 3. Tabaco. 4. Raza blanca. Gráfico de respuestas

1. 2. 3. 4.

Comentario Pregunta muy fácil sobre la osteoporosis en el climaterio y sus factores de riesgo. Se puede responder de forma directa, señalando como opción correcta la 1, ya que las mujeres con obesidad, al tener mayor panículo adiposo, poseen mayor cantidad de aromatasa, que es la enzima encargada de la conversión periférica de los andrógenos en estrona (estrógeno principal en el climaterio). Por ello, la obesidad puede considerarse incluso como un factor de protección. La raza blanca está más predispuesta a padecer osteoporosis (opción 4), al igual que las fumadoras (el tabaco produce hipoestronismo, opción 3), y las multíparas (opción 2) relacionado con el estado estrogénico.(R/1)

32. ¿Cuál es la vía de diseminación más frecuente del cáncer de ovario?: 1. Implantación directa en cavidad abdominal. 2. Vía linfática. 3. Vía hematógena. 4. Transporte a través del tracto genital. Gráfico de respuestas

1. 2. 3. 4.

Comentario La vía de diseminación más frecuente del cáncer de ovario es por contigüidad; en este caso, implantación directa en la cavidad abdominal.(R/1)

33. Un recién nacido a término nacido por cesárea presenta dificultad respiratoria, taquipnea e imagen de "redistribución" de los vasos pulmonares, diafragmas aplanados y líquido en las cisuras. No hay hipoxemia, hipercapnia, ni acidosis; ¿cuál es el proceso más probable?

! !

! 1. Enfermedad de la membrana hialina. 2. Neumomediastino. 3. Taquipnea transitoria del recién nacido. 4. Síndrome de Wilson-Mikity. Gráfico de respuestas

1. 2. 3. 4.

Comentario La taquipnea transitoria es la modalidad de dificultad respiratoria típica del RN término o casi término. Para su génesis, se requiere además un desencadenante obstétrico (parto por cesárea o vaginal rápido), que hace que el tórax del RN no sufra compresión en el canal del parto, con lo que el líquido pulmonar no sería expulsado. Precisamente, ese líquido pulmonar en la radiología y la evolución favorable dan la clave para llegar al diagnóstico. Respuesta 3 correcta.(R/3)

34. A 5-year-old badly behaved child is brought to the pediatrician's office by his mother, because he has very poor school performance. She has noticed that often the boy stares blankly at the wall for a few minutes. He has a normal physical and neurological development. What test should be performed next? 1. EEG study. 2. Holter ECG study. 3. Serological blood tests. 4. Lumbar puncture. Gráfico de respuestas

1. 2. 3. 4.

Comentario EEG study. The reported symptoms are compatible with absence seizures. Absence seizures are defined by a brief loss and return of consciousness, generally not followed by a period of lethargy. In this case, consciousness gaps are responsible for poor school performance. Valproic acid is an effective treatment. A complete medical history and physical examination is the most important diagnostic tool, but EEG is also useful.(R/1)

35. Ante una imagen mamográfica de microcalcificaciones agrupadas y moldeadas, sin tumoración mamaria palpable, ¿cuál es el diagnóstico más probable?: 1. Tumor filodes. 2. Fibroadenoma. 3. Carcinoma intraductal. 4. Carcinoma intralobulillar infiltrante. Gráfico de respuestas

1. 2. 3. 4.

Comentario Pregunta sumamente importante para el ENARM. Lo más importante de esta pregunta es que los signos mamográficos descritos le hagan sospechar un cáncer de mama. La presencia demicrocalcificaciones agrupadas nos obliga a descartar este diagnóstico, por lo que habrá que obtener una muestra histológica. Observe que en el enunciado dicen: “sin tumoración mamaria palpable”. Si hubiese un nódulo claramente definido, la forma de obtener la muestra podría ser mediante una PAAF… Pero si no se palpa nódulo alguno, realizar una PAAF sería como puncionar a ciegas. Por ello, en un caso así, habría que emplear una biopsia guiada con arpón.(R/3)

! !

! 36. Mujer de 32 años de edad con AP de trastorno bipolar en tratamiento crónico con litio. Ingresa en psiquiatría con síntomas de fase maniaca, aunque refiere buen cumplimiento terapéutico. Durante el ingreso se cursa interconsulta a nefrología por poliuria de 7-8 litros al día con polidipsia y nicturia. Exámenes de laboratorio sin alteraciones. Osmolaridad en orina 140 mosm/l. La sospecha diagnóstica y la actitud más adecuadas serán: Diabetes insípida por litio. No precisa pruebas diagnósticas de confirmación; se debe suspender cuanto antes tratamiento con litio. 2. 2. Potomanía. No precisa otras pruebas diagnósticas ni actitud terapeútica alguna. Probable tubulopatía por ingesta crónica de litio. Está indicado realizar un test de 3. 3. deshidratación. Tubulopatía proximal por litio. Suspender tratamiento con litio tras descartar 4. 4. potomanía con un test de deshidratación. Gráfico de respuestas 1.

1.

Comentario Es importante recordar que la ingesta crónica de litio se encuentra entre las causas más frecuentes de diabetes insípida nefrogénica adquirida. En este caso concreto y dado que la paciente presenta descompensación de su trastorno psiquiátrico de base, estaría indicado hacer el diagnóstico diferencial con la potomanía, haciendo un test de deshidratación. (R/3)

37. A 2-day-old infant has a markedly increased cranial perimeter (30 cm). He weighs 2400 gr and thorough physical examination reveals: multiple bilateral cervical and inguinal lymphadenopathies, a bulging fontanel, hepatosplenomegaly and opacities in both eyes. Cardiac auscultation shows a systolic murmur in the second left intercostal space. His mother had a febrile episode during her first trimester of pregnancy but she did not seek medical advice. What is the most likely diagnosis? 1. Perinatal CMV infection. 2. Congenital rubella. 3. Birth-acquired herpes. 4. Congenital Toxoplasmosis. Gráfico de respuestas

1. 2. 3. 4.

Comentario Ante un neonato que presenta antecedente de retraso del crecimiento intrauterino, hiperplasia retículo-endotelial (hepatoesplenomegalia, adenopatías), anemia y trombopenia, debemos considerar una posible infección connatal. Este niño presenta patología ocular (leucocoria, probablemente por cataratas), cardiopatía (soplo) y microcefalia. De los tres datos que integran la tríada de Gregg (cardiopatía, cataratas y sordera neurosensorial), nos han mencionado dos, y el resto del cuadro es compatible, por lo que el diagnóstico más probable corresponde a una rubéola congénita.(R/2)

38. En el desarrollo de venas varicosas en la extremidad inferior el factor MENOS importante es: 1. Antecedentes familiares positivos. 2. Arteriosclerosis obliterante. 3. Edad. 4. Sexo. Gráfico de respuestas

1. 2. 3. 4.

! !

! Comentario Pregunta de dificultad moderada. Estas venas pueden ser primarios o secundarias. Las primarias resultan de un desarrollo defectuoso de las paredes venosas con debilidad congénita de las mismas. Las varices secundarias se producen por insuficiencia venosa profunda y por incompetencia de las venas perforantes. La etiología del proceso varicoso no está clara, aunque se conocen una serie de factores que predisponen y favorecen la aparición de las mismas. El factor más importante es la postura. Aproximadamente la mitad de los pacientes con varices tienen antecedentes familiares. Otros factores son aquellos que dificultan el flujo del retorno venoso como el embarazo, la obesidad o las masas tumorales. Las varices aumentan con la edad y son mucho más frecuente en mujeres.(R/2)

39. Un niño acude a consulta de control al mes de edad y tiene las siguientes medidas: talla 52 cm, peso 4 Kg y perímetro cefálico 37 cm, correspondiendo dichas medidas al percentil 40. Si el niño recibe lactancia materna exclusiva y es un niño sano, usted esperaría en el siguiente control mensual: 1. Que solo la talla persista en el percentil 40. 2. Que el peso suba al percentil 90. 3. Que todas sus medidas continúen en el percentil 40. 4. Ninguna de las anteriores. Gráfico de respuestas

1. 2. 3. 4.

Comentario Es un paciente que se encuentra dentro de la normalidad, por lo que en el control mensual siguiente continuará en dicho percentil.(R/3)

40. Una paciente de 30 años, con un cuadro clínico de sangrado menstrual irregular, dismenorrea, dispareunia y una esterilidad de 3 años de evolución, presenta exámenes de laboratorio hormonales normales y un ultrasonido transvaginal que informa de un útero normal y múltiples formaciones quísticas ováricas bilaterales de 4 cm con signos de sospecha. Teniendo en cuenta el diagnóstico más probable, ¿cuál de las siguientes es de elección para obtener el diagnóstico de sospecha?: 1. Ultrasonido abdominal. 2. TC abdominopélvica. 3. Laparoscopia. 4. Culdocentesis. Gráfico de respuestas

1. 2. 3. 4.

Comentario Ante un posible caso de endometriosis, dada la edad de la paciente y la sintomatología, en cuyo estudio, no tenemos las cosas del todo claras, con unos datos hormonales que no están alterados,

! !

! lo mejor que podemos plantear es la laparoscopía exploradora, para realizar un diagnóstico de certeza y podremos tomar biopsias para un posterior estudio anatomopatológico. (R/3)

41. Paciente con antecedente de alcoholismo que ingresa en urgencias por melenas de 5 días de evolución. En la endoscopía se observan várices esofágicas y gástricas grado III con signos de hemostasia reciente. En un ultrasonido abdominal se aprecia una masa en cabeza de páncreas compatible con pancreatitis crónica que se confirma en el TAC. Un estudio de las presiones portales demuestra que existe hipertensión portal no dependiente del hígado. Para evitar nuevos episodios de sangrado digestivo, recomendaría: 1. Propranolol. 2. TIPS. 3. Resección pancreática. 4. Derivación portosistémica quirúrgica. Gráfico de respuestas

1. 2. 3. 4.

Comentario Nos encontramos ante el caso clínico de un paciente con antecedentes de alcoholismo, sin diagnostico previo de hepatopatía o pancreatitis crónica, que debuta con un episodio de hemorragia digestiva alta y en cuyo estudio descubrimos la existencia de várices esofágicas y gástricas. Lo habitual sería atribuir el origen de éstas a una hepatopatía evolucionada con hipertensión portal. Sin embargo, el estudio posterior demuestra que ésta no depende del hígado y, por otra parte, existe una masa en cabeza de páncreas que provoca hipertensión portal (probablemente por compresión a nivel de la porta). Es decir, tenemos una causa orgánica que justifica el origen de la hipertensión portal, y que por otro lado no podemos descartar que sea una neoplasia. Así pues, nos decidiremos por una técnica quirúrgica resectiva.(R/3)

42. Una de las siguientes entidades se asocia a fibrosis retroperitoneal, mediastínica y colangitis esclerosante: 1. Tiroiditis de Riedel. 2. Tiroiditis de Hashimoto. 3. Carcinoma papilar de tiroides. 4. Carcinoma medular de tiroides. Gráfico de respuestas

1. 2. 3. 4.

Comentario Las tiroiditis son procesos en los que existe una inflamación de la glándula. En muchas de ellas el patrón funcional es de una primera fase de hipertiroidismo por liberación de hormonas tiroideas preformadas al torrente sanguíneo y una segunda fase de hipotiroidismo que puede ser autolimitada. La tiroiditis fibrosante o de Riedel consiste en una fibrosis del cuello que puede afectar al mediastino y al retroperitoneo de forma concomitante, en la que la función tiroidea suele ser normal, existiendo hipotiroidismo hasta en el 25% de los casos, pero el hipertiroidismo no está descrito en esta entidad.(R/1)

43. Una niña de 1 año de edad presenta fiebre alta, vómitos y llanto al miccionar, por lo que se sospecha infección de vías urinarias. En este caso la decisión inicial más adecuada es: 1.

! !

1. Solicitar examen completo de orina tomado con bolsa colectora.

! 2. Pedir urocultivo tomado mediante cateterismo vesical o punción suprapúbica. 3. Indicar ultrasonido de vías urinarias. 4. Iniciar tratamiento antibiótico. Gráfico de respuestas

2. 3. 4.

Comentario Ante la sospecha alta de infección de orina, como en este caso, se puede obviar el sedimento de orina con muestra procedente de bolsa y hacer directamente el sondaje, ya que si el sedimento por bolsa sale alterado, lo que es muy probable, la tendremos que sondar igualmente, y así ganamos tiempo ante una niña con fiebre alta y vómitos, por lo tanto con bastante malestar.(R/2)

44. Entre las contraindicaciones ABSOLUTAS del tratamiento fibrinolítico en el contexto del infarto agudo de miocardio, NO estaría incluido: 1. Antecedentes de hemorragia intracraneal. 2. Hipertensión arterial. 3. Ictus isquémico durante los tres últimos meses. 4. Infarto agudo de miocardio debido a disección aórtica. Gráfico de respuestas

1. 2. 3. 4.

Comentario Contraindicaciones absolutas de tratamiento fibrinolítico: 1. Hemorragia activa (excluida la menstruación). 2. Cualquier hemorragia intracraneal previa. 3. Ictus isquémico en los últimos 3 meses. 4. Lesión cerebrovascular estructural conocida o neoplasia intracraneal. 5. Sospecha de disección aórtica. La hipertensión arterial es una contraindicación relativa, porque podemos intentar controlarla previamente al tratamiento de reperfusión.(R/2)

45. ¿Cuál es el agente causal más frecuente de bronquiolitis en los lactantes?: 1. Virus sincitial respiratorio. 2. Virus de la gripe. 3. Streptococcus penumoniae. 4. Adenovirus. Gráfico de respuestas

1. 2. 3. 4.

Comentario Esta pregunta sobre la etiología de la bronquiolitis es muy sencilla e importante. Recuerde que el virus respiratorio sincitial es el agente causal principal , aunque otros virus como el virus de la parainfluenziae, adenovirus, influenza, rinovirus y, de forma infrecuente, M. pneumoniae, se han asociado a esa enfermedad.(R/1)

46. Embarazada de 37 semanas a la que se le practica monitorización fetal no estresante con el siguiente resultado: frecuencia cardíaca fetal 135 lpm, ondulatoria normal, movimientos fetales escasos, sin ascensos en la frecuencia cardíaca y sin deceleraciones. ¿Qué actitud tomaría? 1. 2. 3. 4.

! !

1. 2. 3. 4.

Inducción del parto. Amnioscopía. Amniocentesis tardía. Prueba de Pose.

! Gráfico de respuestas

Comentario Para valorar el bienestar fetal en el tercer trimestre (37 semanas de gestación tenemos varias opciones, pero en este caso la más acertada será realizar un RCTG estresante o prueba de Pose. •! •! •!

No estaría indicado una inducción de parto ni una cesárea urgente, dado que todavía no queda claro si existe sufrimiento fetal. No estaría indicado realizar una amnioscopía dado que existe riesgo de rotura de membranas y en principio no tenemos intención de inducir el parto. La amniocentesis tardía se suele realizar en la semana 32 para medir la madurez pulmonar fetal, medir la bilirrubina en sospecha de isoinmunización Rh, evacuadora en caso de polihidramnios o amnioinfusión en oligoamnios o tratamiento de la transfusión feto-fetal de gemelares. Este tipo de amniocentesis no se usa por tanto para el diagnóstico prenatal.(R/4)

47. Mujer de 70 años que desde hace tres meses presenta vómitos y dolor abdominal que no cede con la comida. Se acompañaba de pérdida de peso y anorexia. A la EF: palidez mucocutánea y ligero dolor a la palpación epigástrica. En los exámenes de laboratorio se detecta Hb 8.9 g/L, VCM 70, amilasa 45 u/L. El test del aliento para H. pylori fue positivo. En el ultrasonido abdominal se observaron adenopatias en territorio gástrico. Para esta paciente, ¿cual de las siguientes pautas recomendaría en primer lugar?: 1. Gatroduodenoscopia. 2. Pantoprazol 8 semanas y posterior evaluación. 3. PAAF de adenopatias dirigido por TAC. 4. Anticuerpos anti H. pylori. Gráfico de respuestas

1. 2. 3. 4.

Comentario En este caso, con un síndrome general evidente, dolor en epigastrio y anemia microcítica, la primera sospecha diagnóstica debe ser una tumoración gástrica. Por tanto, es necesaria una prueba de imagen (gastroduodenoscopia) y, posteriormente, una confirmación histológica, con biopsia de la lesión. Tanto por la edad de la paciente, como por la positividad del test del aliento para Helicobacter pylori, lo más probable es que se trata de un adenocarcinoma gástrico de tipo intestinal.(R/1)

48. Femenino de 60 años de edad, que presenta sangrado genital escaso por orificio cervical, con ecografía que indica hiperplasia endometrial. ¿Cuál será la conducta más apropiada? 1. Reevaluación ecográfica. 2. Histeroscopia - biopsia. 3. Laparotomía. 4. Histerectomía abdominal. Gráfico de respuestas

1. 2. 3. 4.

Comentario

! !

! La hiperplasia endometrial es un trastorno que consiste en la proliferación del endometrio por acción de los estrógenos sin el efecto compensador de la progesterona. Es una enfermedad que se da fundamentalmente en mujeres con ciclos anovulatorios. El diagnóstico es anatomopatológico. Ante la sospecha de patología endometria por la presencia de clínica (metrorragia perimenopáusica o posmenopáusica) o por hallazgos ecográficos (engrosamiento endometrial mayor de 5 mm en mujeres posmenopáusicas o superior a 12 mm en mujeres premenopáusicas), es obligatorio obtener matieral para estudio anatomopatológico. El método diagnóstico de elección es la histeroscopia-biopsia, ya que ofrece la ventaja de visualizar la cavidad a la hora de realizar la biopsia. Otros métodos también útiles son el legrado fraccionado o las cánulas flexibles tipo Cornier o Pipelle. Entre las opciones disponibles, la más adecuada es la opción 2, siendo la 3 y la 4 demasiado invasivas sin haber obtenido un diagnóstico previo.(R/2)

49. En cuanto al manejo del recién nacido prematuro, señale lo INCORRECTO: El uso de bicarbonato en prematuros extremos condiciona la presentación de hemorragia intracerebral. Los recién nacidos prematuros requieren menores volúmenes de líquidos para 2. 2. infusión intravenosa que los recién nacidos a termino. 3. Una natremia de 150 mEq/L sugiere disminuir la infusión de líquidos. 3. 4. Todos los recién nacidos de bajo peso son prematuros. 4. Gráfico de respuestas 1.

1.

Comentario No todos los RN con bajo peso son prematuros Se ha asociado a la hemorragia intraventricular con el uso de bicarbonato, por lo que la opción 1 no la debe de seleccionar. Por obvias raones en una hipernatremia el tratamiento es diinuir la infusión de suero fisiológico e iniciar glucosado o al 0.45%.(R/4)

50. Paciente de 34 años con antecedentes personales de cólicos biliares que se encuentra en el 7º mes de gestación y que acude a urgencias por presentar dolor abdominal localizado en cuadrante superior derecho de aproximadamente 24-36 h de evolución, tras el cual ha presentado dos vómitos, y que actualmente presenta fiebre de 38ºC. Tras ser valorada por Ginecología sin apreciarse patología ginecológica es remitida a su servicio de urgencias. ¿Qué actitud diagnóstica seguiría?: 1. Exploración física. Exámenes de sangre y Rx de abdomen. 2. Exploración física. Exámenes de sangre y orina y Rx de abdomen. 3. Exploración física. Exámenes de sangre y orina y ultrasonido abdominal. 4. Exploración física y Exámenes de sangre y orina. Gráfico de respuestas

1. 2. 3. 4.

Comentario

! !

! Pregunta deducible, que debe contestar porque aunque dude con el diagnóstico hay varias opciones que puede descartar y por tanto: hay que arriesgarse!. Para contestar bien esta pregunta debe recordar que la urgencia quirúrgica más frecuente en embarazadas es la apendicitis aguda. No le debe despistar la localización del dolor, ya que a medida que progresa la gestación el apéndice se localiza en una posición más alta y más lateral, lo cual puede dificultar el diagnóstico. La apendicitis es un diagnóstico fundamentalmente clínico y se apoya en exámenes de sangre (en la que lo más frecuente es encontrar una leucocitosis con desviación izquierda). En caso de una mujer embarazada, por ser el diagnóstico dudoso suele estar indicado la realización de una técnica de imagen como es un ultrasonido que es inocuo tanto para la madre como para el feto. La Rx de abdomen y el TAC NO estan indicada en embarazadas.(R/3)

51. Cuál de los siguientes enunciados, NO es criterio para considerar que un neonato presenta ictericia fisiológica: 1. Incremento diario de bilirrubina > 5 mg/dl. El nivel máximo de bilirrubina en el RNT se da entre el 2-3 día de vida y en el 2. 2. RNPT entre el 4-6 día de vida. 3. La bilirrubina indirecta no debe llegar en ningún momento a ser > 15 mg/dl. 3. 4. Una de las causas es el aumento de la circulación enteropática de bilirrubina. 4. Gráfico de respuestas 1.

Comentario Las características de la ictericia no fisiológica son las siguientes : Inicio en las primeras 24 horas de vida. Duración superior a 10-15 días. Bilirrubina total mayor de 12 mg/dl en RNT o mayor de 15 mg/dl en RNPT. Incremento de la bilirrubin a superior a 5 mg/dl/24 horas. Bilirrubina directa mayor a 1 mg/dl o superior al 20% de la bilirrubina total.(R/1)

52. Referente a la crioglobulinemia mixta esencial. Todo es cierto, EXCEPTO: 1. Se relaciona con crioglobulinas IgG e IgM monoclonal. 2. Puede ocasionar GN rápidamente progresiva. 3. Los niveles de complemento son normales. 4. La erradicación del proceso infeccioso es de utilidad en el tratamiento. Gráfico de respuestas

1. 2. 3. 4.

Comentario Esta pregunta es de dificultad media. La opción 1 es correcta y no debe dudarla. La clínica renal aparece en un 50% de los casos en forma de hematuria y/o proteinuria, aunque también puede ocasionar una GN rápidamente progresiva (opción 2). La opción 4 es correcta porque se ha demostrado que el tratamiento de la hepatitis puede ayudar al tratamiento de la vasculitis. Por último la opción 3 es INCORRECTA porque precisamente los niveles de complemento suelen estar descendidos por la existencia de inmunocomplejos con IgG e IgM que lo activan.(R/3)

! !

! 53. A 71-year-old woman presents to her physician with a 4-month history of asthenia and arthralgia and a 2-month history of nasal congestion and a 2-week history of cough. She is especially worried now because she has been coughing up blood-tinged sputum for the last 4 days. These symptoms are not accompanied by fever, nausea or vomiting. She has hypertension and migraine. Over the past 4 weeks, she has been taking over-thecounter NSAIDs for her arthralgia. Her pulse is 90/min, breathing frequency are 12/min and blood pressure is 140/83 mm Hg. On physical examination, the patient appears fatigued, with diffuse tenderness in her joints which are not warm or erythematous. A clear nasal discharge is also noted, with no other significant findings. Lab tests show: Hematocrit 32%, Mean corpuscular volume 82 μm3, Leukocyte count 15,100/mm3, Segmented neutrophils 71%, Eosinophils 2%, Lymphocytes 15%, Monocytes 12%, Platelet count 325,000/mm3, Serum Urea nitrogen 32 mg/dL, Creatinine 3.4 mg/dL, Antinuclear antibodies 1:256, Rheumatoid factor negative, Antineutrophil cytoplasmic antibodies positive. Urinalysis shows: Blood 3+, Protein 3+, RBC 15–19/hpf, WBC 1–5/hpf, RBC casts rare. Which of the following is the most likely causative mechanism of this patient's renal failure? 1. Tuberculosis. 2. Interstitial nephritis. 3. Hypertension. 4. Vasculitis. Gráfico de respuestas

1. 2. 3. 4.

Comentario Vasculitis. The combination of pulmonary (blood-tinged sputum), upper airway, arthralgia and kidney failure in a patient with anti-neutrophil cytoplasm antibodies (ANCAs) is suggestive of VASCULITIS, particulary GRANULOMATOSIS WITH POLYANGIITIS a.k.a WEGENER'S DISEASE (if the antibodies react against proteinase= c-ANCA) or CHURG STRAUSS (if the antibodies are pANCA). Remember that the presence of such antibodies is not always detected in patients with these vasculitides.(R/4)

54. Estamos estudiando a una paciente de 45 años, con antecedente personal de vitíligo, que en un control de laboratorio de rutina ha presentado cifras de GOT de 75 U/ml y GPT de 102 U/mL; inmunoglobulina G 1,800mg/dL; serología para hepatitis víricas tipo B y C negativas. La paciente no refiere hábitos tóxicos. Se realiza estudio de anticuerpos que nos pone sobre la pista de una hepatitis autoinmune, que se confirma mediante biopsia. En relación con la hepatitis autoinmune, indique cuál de las siguientes le parece CORRECTA: 1. Es una enfermedad benigna con escasa morbilidad. La hepatitis tipo I, además de presentarse con títulos de anticuerpos antinucleares 2. 2. elevados, presenta serología positiva para el virus C. 3. 3. Los autoanticuerpos anti-LKM-2 se presentan en el tipo II. Los mecanismos inmunitarios humorales parecen los responsables de las 4. 4. manifestaciones extrahepáticas de la enfermedad. Gráfico de respuestas 1.

Comentario La hepatitis autoinmune tipo 1 cursa con ANA y anti-músculo liso positivos. En la tipo 2, aparecen anticuerpos anti- LKM 1 y anti-citosol hepático 1. No obstante, aunque no conozca estos detalles, la respuesta 4 resulta bastante lógica. Si se trata de una enfermedad autoinmune en la que se

! !

! implican autoanticuerpos, resulta lógico pensar que las manifestaciones a distancia tendrán alguna relación con este mecanismo.(R/4)

55. Which of the following statements is CORRECT in relation to iron metabolism? 1. Iron absorption occurs in the ileum. 2. Most of the serum iron is bound to ferritin. 3. Food intake worsens absorption of oral iron. 4. Approximately 20% of the ingested iron is absorbed. Gráfico de respuestas

1. 2. 3. 4.

Comentario La absorción del hierro se realiza en el duodeno y los tramos proximales del yeyuno, y se ve favorecida por el medio ácido. De modo que, con la digestión de los alimentos que aumenta el pH o con fármacos antiulcerosos, la absorción está disminuida. Además, la proporción de hierro procedente de la dieta que se absorbe es de un 10%.(R/3)

56. ¿Cuál de los siguientes esteroides tópicos es el de mayor potencia?: 1. 17 alfa hidrocortisona. 2. Propionato de clobetasol. 3. Butirato-propionato de fluometasona. 4. Valerato de Betametasona. Gráfico de respuestas

1. 2. 3. 4.

Comentario Pregunta compleja, de estudio en profundidad de Segunda Vuelta. Los Esteroides Tópicos, en Dermatología, se dividen en tres grupos según su potencia. Muy potentes: tratamientos en palmas y plantas y dermatosis crónicas, hiperqueratósicas. El más potente de todos es el Propionato de clobetasol (respuesta 3 correcta) y otros menos potentes, también incluidos en este grupo, como el Dipropionato de beclometasona. Moderadamente potentes: los más utilizados. Por ejemplo, Butirato- propionato de fluometasona, Valerato de betametasona, Acetónido de flucinolona. Poco potentes: utilizados en niños y zonas de piel fina, cara, pliegues. El prototipo es la Hidrocortisona.(R/2)

57. After slight pain in his right eye for two days, an otherwise asymptomatic 28-year-old man comes to the emergency department. He bought steroids eye drops at the pharmacy because his eye itched, using it as required during the previous week. After staining with fluorescein, examination reveals a branch-like lesion in lower central cornea. Which of the following is the most appropriate treatment for this patient’s eye lesion? 1. Maintain steroid eye drops treatment until symptoms cease. 2. Add pilocarpine eye drops to the treatment with steroids. 3. Start acyclovir eye drops therapy. Suspend treatment with steroids. Start with acyclovir and pilocarpine eye drops 4. 4. therapy. Gráfico de respuestas 1. 2. 3.

Comentario ! !

! La presencia de una imagen fluoropositiva de forma ramificada es muy sugestiva de que el paciente presente una queratitis herpética. Esta posibilidad se ve muy reforzada por el hecho de que el paciente se haya aplicado un corticoide tópico durante los días previos. Los corticoides por su efecto inmunosupresor pueden favorecer la reactivación del virus herpes, y por ello están contraindicados en esta paciente. También está contraindicada la pilocarpina porque al inducir un espasmo del músculo esfínter de la pupila, acentúa el dolor. (R/3)

58. En una TAC abdominal realizada a una paciente de 42 años para estudio de un traumatismo se detecta una imagen de masa renal derecha de 2.5 cm de diámetro máximo y aspecto graso. Refiere encontrarse asintomática. El resto de la TAC es estrictamente normal. Señale la FALSA: Puede aparecer de forma esporádica, lo cual es cuatro veces más probable en las mujeres. Surge en la esclerosis tuberosa, en cuyo caso es múltiple, bilateral, de mayor 2. 2. tamaño y con probabilidad de causar hemorragias espontáneas. 3. La biopsia es fundamental en el diagnóstico. 3. Las principales complicaciones del AML renal son la hemorragia retroperitoneal 4. 4. y la hemorragia en el sistema colector urinario. Gráfico de respuestas 1.

1.

Comentario El angiomiolipoma (AML) es un tumor mesenquimatoso benigno. Representa en torno al 1 % de los tumores extirpados quirúrgicamente. La ecografía, TC y RM confirman a menudo el diagnóstico debido a la presencia de tejido adiposo. La biopsia rara vez resulta útil. El resto de las opciones otorgan datos fundamentales para el estudio.(R/3)

59. Señale la respuesta FALSA respecto a la fisiología renal: Aclaramiento de una sustancia es el volumen de plasma que se limpia de una sustancia determinada al pasar por el riñón en la unidad de tiempo. 2. 2. Es normal una proteinuria de hasta 150 mg/día. 3. 3. La aldosterona actúa fundamentalmente en el túbulo proximal. 4. 4. Una FENa menor de 1 indica fracaso renal funcional. Gráfico de respuestas 1.

1.

Comentario Esta pregunta entraña una cierta complejidad, pero debe dominar todos estos conceptos para defenderse con soltura ante las preguntas de nefrología. Analicemos cada una de las opciones: 1. La definición de aclaramiento de una sustancia corresponde a lo que dice el enunciado. 2. La proteinuria tiene esos límites antes de ser patológica: 150 mg/día. 3. Intenta aprender la localización de los principales transportadores de iones en la nefrona, tanto para entender el funcionamiento de ciertos diuréticos como la fisiopatología de algunas enfermedades, las tubulares hereditarias y otras. Si es necesario, haga un dibujo de una nefrona y píntelos. Es en el túbulo distal donde actúan tanto la ADH como la aldosterona, regulando la volemia y el agua eliminada. La aldosterona estimula un transportador que reabsorbe Na+ y elimina K+ e H+, ése es el transportador que está alterado en la enfermedad de Liddle.

! !

! 4. La excreción fraccional de sodio (aclaramiento de sodio dividido entre el de creatinina ) menor de uno indica que el fracaso renal es funcional, en esos casos el organismo apenas aclara sodio, intenta reabsober todo el sodio posible para así aumentar la volemia.(R/3)

60. ¿Qué actitud adoptaría ante un paciente ingresado por presentar un infarto de miocardio y que presenta una taquicardia ventricular sostenida, aunque sin deterioro hemodinámico?: 1. Amiodarona. 2. Masaje cardíaco. 3. Bretilio. 4. Verapamil i.v. Gráfico de respuestas

1. 2. 3. 4.

Comentario Se trata de una arritmia ventricular primaria. Si produjera deterioro hemodinámico, al tratarse de una situación de riesgo vital se debería proceder a una desfibrilación eléctrica. Como el estado hemodinámico es bueno, se prefiere intentar primero el tratamiento farmacológico con amiodarona. En caso de que no fuese eficaz, se procedería a la cardioversión eléctrica.(R/1)

61. Un paciente portador de una prótesis mitral metálica va a someterse a una resección prostática. ¿Cuál de las siguientes le parece la profilaxis más adecuada?:* 1. Ampicilina + gentamicina. 2. Ampicilina. 3. Amoxicilina + gentamicina. 4. Ampicilina + metronidazol. Gráfico de respuestas

1. 2. 3. 4.

Comentario En esta pregunta es importante saber que, entre los pacientes que precisan profilaxis de la endocarditis bacteriana ante determinados procesos, se incluyen los portadores de cardiopatías adquiridas, y dentro de ellos, están los que llevan prótesis intracardiacas como la prótesis mitral de este caso clínico. En segundo lugar hay que recordar que la profilaxis en procedimientos genitourinarios es con ampicilina 2g i.v. y gentamicina 1.5mg/kg de peso i.v o i.m. 30 min antes del procedimiento, seguidos de ampicilina 1g, 6 horas después. Para los procesos odontológicos recuerda que la profilaxis se realiza con ampicilina. Aunque recientemente se han publicado las guías americanas, en las que ya no se recomienda profilaxis antimicrobiana en procedimientos gastrointestinales, ni genitourinarios.(R/1)

62. La enterotoxina colérica es responsable del cuadro diarreico característico de esta enfermedad. ¿Cuál es su mecanismo de acción?: 1. Efecto citotóxico directo sobre el enterocito. 2. Provocando la activación continua de la adenilato ciclasa. 3. Por activación del GMP cíclico intracelular. Provocando la liberación de citoquinas por las células del sistema inmune 4. 4. intestinal. Gráfico de respuestas 1. 2. 3.

Comentario ! !

! Pregunta directa sobre el mecanismo fisiopatológico de producción de diarrea del Vibrio cholerae. Vibrio cholerae, agente causal del cólera, es una bacteria productora de enterotoxinas, las cuales actúan directamente sobre la superficie de los enterocitos, sin destrucción de la mucosa, alterando el intercambio iónico y favoreciendo el paso de agua libre hacia la luz intestinal (a través de la activación continua de la adenilato ciclasa) por eso genera tanta deshidratación para ocasionar una diarrea acuosa sin productos patológicos y sin leucocitos al microscopio.(R/2).

63. Ante un paciente con demencia de reciente comienzo y una serología treponémica positiva en sangre, ¿qué prueba complementaria considera esencial para establecer el diagnóstico de neurosífilis? 1. EEG. 2. RM craneal. 3. TC craneal con contraste. 4. Análisis del LCR. Gráfico de respuestas

1. 2. 3. 4.

Comentario El VDRL, que es una prueba serológica no treponémica, se utiliza para diagnosticar la neurosífilis. Lógicamente, el estudio se realizará en LCR. Este examen consiste en buscar determinados anticuerpos, llamados reaginas, en el líquido cefalorraquídeo. Cuando se trata de una sífilis primaria o secundaria, esta prueba sería negativa. Sin embargo, en una neurosífilis, el resultado sería positivo. Recuerde que las pruebas serológicas para la sífilis se dividen en dos grupos: Reagínicas: VDRL y RPR, menos sensibles y específicas. Pueden negativizarse con la curación de la enfermedad, por lo que tienen gran valor para el seguimiento. Treponémicas, como el FTA- Abs. Su sensibilidad y especificidad son mayores. Sin embargo, una vez que son positivas, ya nunca negativizan. Por ello, no tienen valor para el seguimiento de un paciente, ni para diagnosticar una reinfección.(R/4)

64. La operación de Whipple consiste en la exéresis de: Parte distal del estómago, duodeno, cabeza de páncreas, vesícula, vía biliar distal y yeyuno proximal. 2. Vesícula, vía biliar distal y cabeza de páncreas. 2. 3. Vesícula, vía biliar distal y parte distal gástrica. 3. 4. Vesícula, vía biliar proximal y distal y cabeza de páncreas. 4. Gráfico de respuestas 1.

1.

Comentario Se trata de una pregunta específica sobre técnica quirúrgica y en concreto sobre la intervención de Whipple o duodenopancreatectomía cefálica, que es la técnica quirúrgica de elección con intención curativa indicada en el cáncer de cabeza de páncreas. En esta técnica se resecan el antro gástrico, todo el marco duodenal y unos cms del yeyuno proximal en bloque junto con la cabeza del páncreas que se secciona a nivel del llamado cuello pancreático sobre la porta y que incluye la vía biliar principal distal, es decir el colédoco distal. Como gesto añadido se debe resecar la vesícula biliar, lo cual es mandatorio siempre que se practica una intervención en la vía biliar principal. Así

! !

! pues debemos reconstruir haciendo tres anastomosis a saber: 1) anastomosis entre el conducto de Wirsung y el tubo digestivo 2) anastomosis entre la vía biliar y el tubo digestivo y 3) reconstrucción del tránsito digestivo mediante anastomosis entre el estómago y el tubo digestivo. En general todas estas anastomosis se realizan sobre un mismo asa de yeyuno.(R/1)

65. Una paciente, diagnosticada de infección crónica por VHB en fase replicativa y en tratamiento con interferón alfa-2b, consulta porque, a las ocho semanas de iniciado el tratamiento, está más cansada y un estudio de laboratorio demuestra que hay un ligero aumento de la bilirrubina y de las transaminasas. ¿Qué consideraciones se haría? 1.! Probablemente!tiene!una!hepatitis!de!otra!causa.! ! 2.! Se!debe!añadir!esteroides!al!tratamiento.! ! 3.! Probablemente!está!haciendo!la!seroconversión!anti inhalatoria > oral ( opción 1 correcta). El síndrome de abstinencia depende de la dosis, el tiempo de consumo, y la vida media. Concretamente con la heroína aparece a partir de las 8 horas (opción 2 correcta) de la última dosis, con la metadona, al tener vida media más larga, la abstinencia es más larga pero más leve y por eso se utiliza como sustitutivo para la deshabituación. El opiode que más pronto induce abstinencia la meperidina. Para evitar los síntomas de la abstinencia se una clonidina y así disminuye la hiperactividad adrenérgica con agonistas alfa- 2. Lo que es falso es que la naltrexona esté indicada para el tratamiento de la intoxicación aguda. Lo que se usa es la Naloxona! (R/4)

79. Una de las siguientes NO es complicación frecuente del abruptio: 1. Insuficiencia renal. 2. Choque. 3. Síndrome de Sheehan. 4. Pielonefritis. Gráfico de respuestas

1. 2. 3. 4.

Comentario Como complicaciones del abruptio, se han descrito todas las respuestas, excepto la pielonefritis (opción 4). Las más frecuentes son la CID y la insuficiencia renal. La CID también se presenta en abortos diferidos y la patogenia es similar en ambos casos: liberación y paso de tromboplastinas fetales a circulación materna. El tratamiento consiste en una adecuada reposición de factores de coagulación y fibrinógeno, al mismo tiempo procedemos a la evacuación inmediata del útero, mediante la realización de una cesárea. El choque y el síndrome de Sheehan son complicaciones menos frecuentes.(R/4)

80. Niño de 4 años con fiebre alta de 6 días de evolución acompañada de irritabilidad. La madre refiere además haberle notado las manos y los pies hinchados y dolorosos. A la exploración llama la atención hiperemia

! !

! conjuntival intensa sin exudado, un exantema de predominio en tronco y labios enrojecidos y resecos. Es ingresado con sospecha de enfermedad infecciosa y tratado con antibióticos tras la toma de cultivos. Al 10º día de evolución, la fiebre no ha desaparecido y comienza a presentar alteraciones en manos y pies como los que se muestran en la imagen. En relación al cuadro clínico citado, señale la opción FALSA: La imagen muestra la descamación en dedo de guante característica de la enfermedad. Para confirmar el diagnóstico es necesario realizar biopsia de algún ganglio 2. 2. linfático accesible. 3. 3. La base patogénica de la enfermedad es una vasculitis. La etiología de la enfermedad no está clara pero se basa en la teoría del 4. 4. superantígeno. Gráfico de respuestas 1.

1.

Comentario La imagen, en esta pregunta, es sólo un dato más, y nos muestra lo que ya nos podríamos haber imaginado por el contexto de la pregunta: descamación plantar. Se trata de una enfermedad de Kawasaki, en cuya patogenia parece estar implicado un superantígeno que produciría una sobreactivación policlonal de los linfocitos T, que producirían citoquinas responsables de las manifestaciones de la enfermedad. Recuerden que, en esta patología, el problema fundamental es que a veces se producen dilataciones aneurismáticas en las coronarias, por lo que estos niños estarían predispuestos a infartos de miocardio a edades muy precoces. El diagnóstico de la enfermedad de Kawasaki se realiza en base a criterios clínicos y no precisaría confirmación histológica (R/2).

81. El niño de la pregunta anterior fue ingresado y tratado con gammaglobulina intravenosa además de aspirina. Sobre el manejo de este paciente, señale la opción FALSA: Si la fiebre no cede con gammaglobulina el fármaco de segunda elección son los corticoides. La gammaglobulina es eficaz en la prevención de la aparición de aneurismas 2. 2. coronarios en la mayoría de los casos. La aspirina a dosis antiagregantes es importante en la prevención de las 3. 3. trombosis. La descamación en dedo de guante es característica de la fase aguda de la 4. 4. enfermedad y por tanto muy útil para el diagnóstico precoz y el inicio del tratamiento. Gráfico de respuestas 1.

1.

Comentario La descamación es habitual cuando la enfermedad ya lleva cierta evolución, por lo que no tendrá aplicación alguna para llegar a un diagnóstico precoz (respuesta 4 falsa). Tal como explicábamos en el comentario anterior, el problema principal que puede complicar este cuadro son los aneurismas coronarios, que aparecen con menor frecuencia cuando se utiliza la gammaglobulina intravenosa, como se comenta en la opción 2.(R/4)

82. El fenómeno de Koebner se da en:

! !

! 1. Pénfigo. 2. Psoriasis. 3. Púrpura trombocitopénica. 4. Vasculitis por hipersensibilidad. Gráfico de respuestas

1. 2. 3. 4.

Comentario Pregunta básica sobre un signo muy útil en el diagnóstico dermatológico, el fenómeno de Koebner. Consiste en la aparición de determinadas dermatosis en las zonas de presión o traumátismo. Es típico de las enfermedades eritemato-escamosas como psoriasis (opción 3), liquen plano, pitiriasis rubra pilaris y Darier. Infecciosas con el molluscum o las verrugas y de otras como vitíligo, xantomas, vasculítis y síndrome de Sweet. La consecuencia práctica es que el paciente debe conocer que el fenómeno puede actuar como factor precipitante, por lo que debe evitar en la medida de lo posible las lesiones cutáneas.(R/2)

83. El tratamiento más recomendable de la diverticulosis asintomática es: 1. Laxantes salinos. 2. Anticolinérgicos. 3. Resección quirúrgica del segmento afecto. 4. Dieta rica en residuos. Gráfico de respuestas

1. 2. 3. 4.

Comentario Cuando se habla de diverticulosis, el término se refiere a la presencia de divertículos, sin complicación acompañante. La diverticulosis aumenta en frecuencia con la edad, y suele ser asintomática. Por tanto, es evidente que, de inicio, no se indica ningún tratamiento quirúrgico. En situación estable, sin clínica, no se precisa el uso de fármacos de ningún tipo. En cambio, sí parece aconsejable una dieta rica en fibra y residuos, que “arrastre” el contenido del colon, lo que dificulta el estancamiento de restos fecales intradiverticulares, lo que incrementa el riesgo de diverticulitis.(R/4)

84. A patient with a past history of HIV infection comes to the emergency department complaining of dyspnea and pallor. Current medication includes antiretroviral therapy with AZT + 3TC + Efavirenz, for 14 months. Last determinations of CD4 and viral load were 350/mm3 and o igual 8.(R/3)

256. Respecto a la enfermedad de Paget de la mama, señale la respuesta FALSA: 1. A diferencia del eccema, presenta límites netos. 2. Se asocia siempre a un adenocarcinoma ductal subyacente. Se caracteriza histológicamente por las "células de Paget", que tiñen con PAS y 3. 3. son diastasa resistentes. 4. 4. No es pruriginosa. Gráfico de respuestas 1. 2.

Comentario Pregunta de dificultad media sobre la enfermedad de Paget de la mama. La enfermedad de Paget debe considerarse como la extensión de un tipo de cáncer de mama ductal a la epidermis suprayacente, afectando pezón y la areola. Es poco frecuente, representando el 1- 3% de todos los cánceres de mama. Clínicamente, consiste en una lesión eccematosa, asociando también prurito (respuesta 4 falsa). Por ello, el eccema es su principal diagnóstico diferencial. Una de las características por las que puede distinguirse del eccema es por ser unilateral (el eccema suele afectar a ambas mamas). Otro rasgo diferencial son los límites, que suelen ser difusos en el eccema y mejor definidos en el Paget mamario. Recuerda que, histológicamente, son características las células de Paget. Son grandes, redondeadas, PAS- positivas, con núcleos de gran tamaño y sin puentes intercelulares que las unan. (R/4)

257. Señale de entre los siguientes antígenos HLA aquellos que estén relacionados estrechamente con la diabetes mellitus tipo 1:

! !

! 1. B27, B29. 2. DR8, DR9. 3. D3, D4. 4. B10, B12. Gráfico de respuestas

1. 2. 3. 4.

Comentario En la diabetes mellitus tipo 1 participan muchos genes y el mecanismo herediartio no está bien conocido. El principal gen de predisposición a la DM-1se localiza en la región HLA del cromosoma 6. la mayoría de los diabéticos tiene el haplotipo HLA DR3, HLA DR4 o ambos. Sin embargo, la mayoría de los individuos portadores de estos haplotipos no padecen diabetes y la mayoría de los individuos afectos de esta enfermedad carece de pacientes de primer grado que la sufran. El DR2 puede tener papel protector. (R/3)

258. El principal factor etiológico del embarazo ectópico sería: 1. La utilización de DIU. 2. Endometriosis. 3. Antecedentes de EIP. 4. Edad materna. Gráfico de respuestas

1. 2. 3. 4.

Comentario Entre los factores de riesgo de embarazo ectópico, encontramos todas las opciones que nos presentan, salvo la edad materna. De ellos, el más frecuente es la presencia o los antecedentes de enfermedad inflamatoria pélvica. La inflamación de las trompas modifica la motilidad de las mismas y produce fenómenos de cicatrización, por lo que predispone al embarazo ectópico. La cirugía abdominal y la endometriosis producirían adherencias que pueden englobar a las trompas y así modificar también su motilidad. El DIU disminuye el embarazo intrauterino, pero no es tan eficaz con los extrauterinos, por lo que aparece un aumento de la frecuencia relativa de los mismos. (R/3)

259. A 9-year-old girl is brought to consultation for a routine visit. She is taller than expected for her age. She has no previous medical history. Physical examination shows mild scoliosis, disproportionately long arms and legs and abnormal joint flexibility. Her father died suddenly and then the postmortem diagnosis of ruptured aortic aneurysm was made. What is the most likely etiology of this condition? 1. Mutations of the fibrillin 1 gene. 2. Vitamin D deficiency. 3. Defective collagen production. 4. Mutations of the procolagen type 1 gene. Gráfico de respuestas

1. 2. 3. 4.

Comentario Mutations of the fibrillin 1 gene. Patients with Marfan syndrome have typical phenotype features, such a high height and joint hyperlaxity. Family history is very useful and cases of sudden death due to aneurysm rupture may be present, as in this case. The disease is caused by a mutation in the fibrillin1 gene. Collagen mutation predispose to osteogenesis imperfecta. (R/1)

260. Un paciente de 27 años viene a la consulta con una historia de 1 año de duración de dolor en ambos glúteos irradiado por cara posterior de muslo hasta su porción media. El

! !

! dolor es peor al amanececer; se acompaña de rigidez lumbar y mejora con el ejercicio. ¿Cuál de los siguientes procedimientos aportaría más información diagnóstica en este paciente? 1. Los niveles de proteína C reactiva y de antiestreptolisina O (ASLO). 2. La resonancia magnética de columna lumbar. 3. El tipaje HLA-B27. 4. La radiografía de las articulaciones sacroilíacas. Gráfico de respuestas

1. 2. 3. 4.

Comentario Cuadro clínico compatible con espondilitis anquilosante: varón joven con afectación de esqueleto axial (dolor en región glútea bilateral típicamente inflamatorio: peor en reposo y que mejora con el movimiento). Dado que la presencia de sacroileítis radiológica es imprescidible para el diagnostico de EA debemos solicitar en primer lugar una Rx de sacroilíacas (opción 4 correcta). La VSG y PCR son inespecíficas, el tipaje HLA- B27 no es necesario para el diagnóstico y los ASLO no se utilizan en la EA. (R/4)

261. ¿Qué afirmación sobre las atresias intestinales es FALSA?: 1. Puede existir antecedente de polihidramnios. 2. La asociación con síndrome de Down es frecuente. La observación de calcificaciones intraperitoneales en la radiografía descarta este 3. 3. diagnóstico. 4. La atresia de colon puede asociarse a enfermedad de Hirschprung. 4. Gráfico de respuestas 1. 2.

Comentario Las atresias intestinales son cuadros muchas veces de tediosa evolución, pues requieren múltiples intervenciones quirúrgicas y ocasionalmente derivan hacia la presencia de secuelas tales como el intestino corto. Son especialmente frecuentes en los niños con síndrome de Down, a nivel de yeyuno-íleon. Más raramente afectan al colon, en el contexto de un megacolon agangliónico extremo. Suele tener antecedente obstétrico de polihidramnios. Si generan peritonitis meconial intraútero, en la radiografía de abdomen se aprecian calcificaciones peritoneales dispersas. (R/3)

262. Un paciente alcohólico de 55 años es llevado a urgencias por la policía. El paciente se halla somnoliento y desorientado. Su aliento huele a alcohol. Las pupilas son isocóricas normorreactivas, y los movimientos oculares, normales. En las maniobras de Barré y Mingazzini se comprueba una paresia de las extremidades izquierdas, y al explorar el reflejo cutáneo plantar, se halla un Babinski izquierdo. ¿Cuál de los siguientes enunciados es correcto? Nos hallamos únicamente ante un caso de intoxicación aguda enólica y no debemos realizar ninguna exploración complementaria. 2. 2. El cuadro es específico de encefalopatía de Wernicke y se administrará B1. 3. El paciente sufre una polineuritis aguda y se practicará una punción lumbar. 3. Debe practicarse una TC craneal o una angiografía cerebral para descartar un 4. 4. accidente vascular cerebral. Gráfico de respuestas 1.

1.

Comentario ! !

! La presencia de focalidad neurológica, con afectación de predominio crural y signo de Babinski positivo (vía piramidal) orienta fuertemente hacia la respuesta 4. El resto de las opciones se descartan con facilidad. La encefalopatía de Wernicke obedece a un déficit de tiamina y se caracteriza por la tríada: oftalmoparesia, ataxia y síndrome confusional. Las polineuropatías son cuadros de instauración gradual, normalmente simétricos y de predominio distal. Por último, la intoxicación alcohólica aguda tampoco explica esta clínica (es posible que también la tenga, porque huele a alcoho, pero no justifica la paresia ni el Babinski). (R/4)

263. Son signos clínicos de choque hipovolémico temprano, los siguientes EXCEPTO: 1. Taquicardia. 2. Cianosis. 3. Estado mental alterado. 4. Presión arterial normal. Gráfico de respuestas

1. 2. 3. 4.

Comentario Debes dominar la siguiente tabla, para fines del ENARM y la vida. La cianosis es una manifestación clínica que aparece en etapas más avanzadas del proceso. (R/2)

264. Paciente de 42 años con carcinoma de cérvix, diagnosticado por biopsia, que invade el labio anterior y 1 cm de la cúpula vaginal; los parametrios no están invadidos; la

! !

! rectoscopía, la urografía descendente y la cistoscopía son normales. En este caso, el tratamiento más CORRECTO sería: 1. Radioterapia externa más quimioterapia. 2. Histerectomía total ampliada tipo Wertheim-Meigs más radioterapia externa. 3. Radioterapia de contacto, seguida de histerectomía total simple. 4. Ninguna de las anteriores. Gráfico de respuestas

1. 2. 3. 4.

Comentario Pregunta teórica acerca del tratamiento del carcinoma de cérvix, debiendo conocer datos básicos del TNM. Recuerde que el estadio IIB implica afectación de parametrios. En este caso nos presentan un carcinoma cervical estadio IIA por lo que el tratamiento de elección es la histerectomía radical con linfadenectomía pélvica (cirugía de Wertheim-Meigs). La única opción que te ofrece esta posibilidad es la respuesta número 2 (con RT externa adyuvante).

265. A 27-year-old woman that had an uncomplicated vaginal delivery three days ago comes to the doctor's presenting with asthenia. Serologic test results are HBsAg Positive, Anti HBsAg negative, IgM Anti HBcAg Positive, HBe Ag positive, Anti HBe Ag negative. What is the best next step in the management of the newborn? 1. Administration of hepatitis B immune globulin followed by vaccination. 2. Hepatitis B vaccination. 3. Initiate treatment with lamivudine. 4. Periodic serologic control. Gráfico de respuestas

1. 2. 3. 4.

Comentario Administration of hepatitis B immune globulin followed by vaccination. Routine serological tests are performed during pregnancy in order to treat connatal infection. In the case of VHB infection there are measures of proved efficacy: at birth, administration of intravenous Ig and the first vaccination dose.(R/1)

266. Una de las siguientes afirmaciones es típica de la artrosis primaria: El recuento celular del líquido sinovial es con frecuencia superior a 3,000 células por milímetro cúbico. 2. Afecta principalmente a las articulaciones metacarpofalángicas. 2. 3. En la radiografías se aprecia osteoporosis. 3. 4. La viscosidad del líquido sinovial es elevada. 4. Gráfico de respuestas 1.

1.

Comentario El líquido sinovial NO es inflamatorio, es un líquido de viscosidad normal (esto es, viscosidad no disminuída), con recuento de células, glucosa y proteínas dentro de la normalidad. Tampoco los exámenes de laboratorio ayudan al diagnóstico, pues los reactantes de fase aguda, BH y bioquímica son completamente normales. Así, pues, la clínica es la base del diagnóstico, y los cambios radiológicos lo más importante a conocer. Las articulaciones más afectadas son la cadera, la rodilla, la columna y las articulaciones interfalángicas de manos y piés (recuerde que la afectación de MCF es propia de la artritis reumatoide). En la radiografía habrá pinzamiento del

! !

! espacio articular, esclerosis subcondral, osteofitos, quistes yuxtaarticulares y deformidad articular.(R/4)

267. Ante un paciente con una faringitis gonocócica, ¿cuál de los siguientes tratamientos NO es eficaz?: 1. Penicilina. 2. Ciprofloxacina. 3. Ceftriaxona. 4. Espectinomicina. Gráfico de respuestas

1. 2. 3. 4.

Comentario Pregunta de dificultad intermedia sobre antibioterapia. El tratamiento de elección del gonococo son las cefalosporinas de tercera generación, como la ceftriaxona (opción 3). El ciprofloxacino es una quinolona y como tales antibióticos de amplio espectro y útiles frente a gramnegativos. La espectinomicina es un antibiótico que aunque no es un aminoglucósido su espectro de acción es parecido, y se extiende a varias bacterias grampositivas y gramnegativas, aunque en la práctica queda limitado a la N. gonorrhoeae. La penicilina en principio es un antibiótico para grampositivos, además muchas cepas del gonococo son productoras de betalactamasas (R/1).

268. Señale la respuesta FALSA en relación a la exploración del pulso venoso yugular: Los pacientes con presión venosa muy elevada deben ser explorados en decúbito supino para que se haga evidente la altura de la columna de presión yugular. Los movimientos de aurícula y ventrículo derechos crean ciertos cambios en el 2. 2. flujo de retorno que se manifiestan por ondas de volumen en las venas principales que abocan al corazón. Las dos ondas positivas (a, v) son visibles y casi coincidentes con los ruidos 3. 3. cardíacos. La exploración se debe hacer de las venas yugulares internas y externas de ambos 4. 4. lados del cuello. Gráfico de respuestas 1.

1.

Comentario Pregunta de baja dificultad de semiología cardíaca, sobretodo el pulso venoso yugular. Esta se mide viendo el latido yugular con el paciente en decúbito y levantado el tórax unos 45º. Se mide la altura con respecto al ángulo de Louis, que está a unos 5 cm de AD. La causa más frecuente de elevación de la presión venosa es un aumento de la presió diastólica ventricular. El pulso venoso yugular consta generalmente de dos ondas positivas ("a" y "v") y dos depresiones negativas "x" e "y". La onda "a" es un onda prsistólica que se debe a la contracción auricular y se sitúa justo antes de R1 mientras que la onda "v" tiene lugar durante la contracción ventricular y ocurre durante R2. (R/1)

269. Señale qué afirmación, de las siguientes, relativa al meconio fetal normal, es CORRECTA: 1. 2.

! !

1. Contiene escasos estafilococos. 2. Excepcionalmente contiene estreptococos.

! 3. Puede contener Proteus en casos de prematuridad. 4. No contiene ningún germen. Gráfico de respuestas

3. 4.

Comentario Pregunta sencilla sobre el meconio. El meconio es estéril, por lo que la respuesta correcta es la número 4.

270. En relación con el cuadro de sepsis neonatal de comienzo tardío, señale cuál de estas afirmaciones NO es correcta: 1. Suele presentarse a partir de la semana de vida. 2. Los agentes etiológicos más frecuentemente implicados suele ser los anaerobios. 3. La clínica es más insidiosa o solapada que la de la sepsis neonatal precoz. El tratamiento de la meningitis nosocomial en un RN ingresado consiste en la 4. 4. administración de vancomicina + cefotaxima. Gráfico de respuestas 1. 2. 3.

Comentario La sepsis neonatal tardía no nosocomial, que aparece a partir del séptimo día de vida, tiene como agentes más frecuentes determinados serotipos de SGB y E. coli. Estos serotipos peculiares tienen como característica esencial la presencia de antígenos de adherencia tisular (por eso las infecciones focales son más frecuentes). Su presentación no es tan abrupta como en las sepsis precoces. Para tratar las meningitis nosocomiales ha de usar vancomicina (para cubrir los estafilococos) y cefotaxima (que cubre bien a los gram negativos), pudiéndose asociar además anfotericina B liposómica. (R/2)

271. ¿Cuál de las siguientes es CORRECTA a cerca de la anemia perniciosa?: La médula generalmente muestra hipocelularidad con discreta elevación en el número de megaloblastos. 2. Es frecuente la presencia anticuerpos anti-celulas parietales. 2. 3. Es característico el aumento de absorción del hierro de los alimentos. 3. La administración de folato es tratamiento alternativo cuando no hay respuesta al 4. 4. tratamieneto con vitamina B12. Gráfico de respuestas 1.

1.

Comentario La anemia perniciosa, es la causa fundamental de anemia megaloblástica por déficit de B12. Consiste en una destrucción autoinmune (presencia de anticuerpos ), de las celulas parietales gástricas productoras de Factor intrínseco, necesario para la absorción de la Vitamina B12, asi como de acido clorhídrico, necesario para la absorción del hierro de los alimentos. El déficit de B12, ocasiona un retardo en la división celular, con eritropoyesis ineficaz y aumento de la celularidad de la medula ósea. La B12 logicamente, no se puede dar VO, si el transtorno consiste en una alteración de su reabsorción. El folato es aconsejable darlo junto la B12, porque se produce también un déficit de este, pero nunca como alternativa. (R/2)

272. Son complicaciones de la obesidad en niños: 1. 2.

! !

1. Alteraciones ortopédicas. 2. Pubertad temprana.

! 3. Desarrollo ulterior de diabetes mellitus tipo 2. 4. Todas las anteriores. Gráfico de respuestas

3. 4.

Comentario Respuesta sencilla sobre complicaciones de obesidad. La respuesta correcta es la 4, ya que la obesidad puede generar alteraciones ortopéidcas, pubertad temprana (por el exceso de grasa, que incrementa los niveles de andrógenos) y el desarrollo ulterior de DM tipo 2.

273. Es una CONTRAINDICACIÓN absoluta para realizar un fórceps: 1. Dilatación completa. 2. Presentación cefálica. 3. Cabeza en II plano. 4. Sufrimiento fetal. Gráfico de respuestas

1. 2. 3. 4.

Comentario El fórceps es un método de extracción fetal que se emplea durante el período expulsivo, conceptualmente semejante a unas tenazas. Exige, por tanto, dilatación completa y bolsa rota. Requiere presentación cefálica (salvo el raro caso del fórceps de cabeza última, en el parto de nalgas complicado) y cabeza normal (no puede usarse en hidrocefalias o anencefalias). Para poderse emplear con seguridad, el punto guía debe alcanzar el tercer plano de Hodge, signo de que el diámetro biparietal ha sobrepasado el estrecho superior. El fórceps es un instrumento rápido, que resuelve malrotaciones fetales (tractor y rotador), lo que le convierte en instrumento ideal en caso de sufrimiento fetal (siempre que se den las condiciones que hagan posible su aplicación).(R/3)

274. Niño de 2 años de edad con fiebre de 39ºC de 48 horas de evolución asociado a cuadro catarral leve. Acude por aparición de vesículas en pilares anteriores faríngeos así como afectación palmo-plantar. Respecto a la patología que sospecha señale la afirmación CORRECTA: 1. 2. 3. 4.

1. Se caracteriza por presentar un exantema muy pruriginoso. 2. Es causada por el virus Coxsackie B. Se debe administrar gammaglobulina a neonatos expuestos en las primeras 72 3. horas. 4. Puede ser causado por Enterovirus.

Comentario El paciente padece la enfermedad mano-pie-boca que se caracteriza por aparición de vesículas en mucosa oral, paladar y pilares anteriores faringeos y vesículas en manos y pies principalmente en el dorso pero también en palmas y plantas. Puede aparecer un exantema maculopapulosos en cara, brazos y nalgas. Es causado por el Coxsackie A16 y enterovirus 71. Respuesta correcta 4.

! !

! Es importante no confundir esta enfermedad con varicela, en esta última se encontrarán lesiones en distitnos estadíos y no habría afección palmo-plantar.

275. Respecto a las complicaciones que pueden producirse en esta patología señale la respuesta INCORRECTA: 1. Meningitis. 2. Encefalitis. 3. Convulsiones febriles. 4. Neumonitis. Gráfico de respuestas

1. 2. 3. 4.

Comentario El enterovirus 71 puede causar meningitis y encefalitis, minetras que el Coxsackie A16 puede causar miocarditis. La enfermedad exantemática que más se ha relacionado con las convulsiones febriles es el exantema súbito.(R/3)

276. Señale cuáles son los elementos del triángulo de Hesselbach. 1. Vasos epigástricos, ligamento de Cooper, borde lateral del recto abdominal. Ligamento inguinal, vasos epigástricos y borde lateral del recto anterior 2. 2. abdominal. 3. Ligamento inguinal, vasos epigástricos, borde interno del recto abdominal.. 3. 4. Vasos epigástricos, ligamento de Cooper, borde interno del recto abdominal.. 4. Gráfico de respuestas 1.

Comentario Pregunta de cirugía en el internado, de anatomía de las hernias inguinales. El triángulo de Hesselbach está compuesto por el ligamento inguinal, los vasoso epigástricos y el borde lateral del recto abdominal. Respuesta 2 correcta.

! !

!

277. Se recibe en consulta a un niño de 5 meses de edad, cuyo signo mayor es la pobre ganancia ponderal. Peso al nacer: 3,700 gr. Peso actual: 4,500 gr. La madre refiere que lacta de manera entrecortada con sudoración excesiva y se muestra irritable. Exámen físico: taquipnea. ¿Cuàl es el diagnóstico más probable? 1. Errores innatos del metabolismo. 2. Mala técnica de lactancia. 3. Infección intrauterina: TORCHS. 4. Insuficiencia cardíaca congestiva. Gráfico de respuestas

1. 2. 3. 4.

Comentario La opción más probable, dado que sólo nos indican que existe una pobre ganancia ponderal (recordamos que a los 5 meses el niño debería de haber duplicado su peso al nacer: 7,400 gr aproximadamente, pesando en la actualidad 4,500 gr) y ningún otro signo clínico o síntoma que nos haga sospechar la opción 1, 3 ó 4, sería la mala técnica de lactancia por la cual el niño se alimenta de forma entrecortada y esto le genera una importante sudoración e irritabilidad. Sería necesario instruir a la madre en una técnica adecuada para proporcionar el alimento a su hijo.(R/2)

278. En el tratamiento de la diabetes mellitus tipo 1 en niños, se tendrá presente lo siguiente, señale lo CORRECTO: 1. La insulina de acción intermedia es de elección para la cetoacidosis. 2. La glicemia debe mantenerse en valores menores de 140mg/dl. 3. La administración de bicarbonato es obligatoria para corregir la cetoacidosis. 4. El horario de las comidas debe guardar relación con el esquema insulínico. Gráfico de respuestas

1. 2. 3. 4.

Comentario La insulina que se utiliza para la corrección de la cetoacidosis es la regular via iv. El bicarbonato sólo se debe administrar cuando el pH es menor de 6.9, sinó, la acidosis se corregirá con la insulina e hidratación administrada. El horario de las comidas se debe coordinar con las comida, para hacer más fisiológica su curva y para que no haya hipoglicemias. (R/4)

279. Un hidradenoma está asociado con: 1. Células nerviosas. 2. La vejiga. 3. Hernias. 4. Glándulas sudoríparas. Gráfico de respuestas

1. 2. 3. 4.

Comentario

! !

! Pregunta de dificultad elevada. El hidradenoma papilar es una neoplasia benigna. Se trata de un nódulo bien delimitado localizado sobre todo en los labios mayores. Puede ulcerarse. Surge a partir de las glándulas sudoríparas apocrinas de la vulva. Se han descrito asociaciones con hernias. Respuesta 3 correcta.

280. Recién nacido de 20 horas de vida, edad gestacional 38 + 1 semana, peso al nacimiento 2,800 gramos y Apgar 9/9. Madre de 35 años (G3 A2 P1), hipotiroidea en tratamiento sustitutivo. El parto ha sido vaginal y eutócico, como antecedentes destaca bolsa rota 5 horas antes del parto. Presenta ictericia conjuntival y cutánea hasta el ombligo. No presenta cefalohematoma, ni otros signos de hemorragia. La palpación de las clavículas es normal, y el cráneo no crepita. Ha recibido 3 tomas al pecho materno. El grupo sanguíneo materno es O- y el del recién nacido es B+. Es FALSO que: Un porcentaje de casos requerirá tratamiento con fototerapia y algunos con exanguinotransfusión según las cifras de bilirrubina. El diagnóstico diferencial habría que realizarlo principalmente entre cualquier 2. 2. forma de isoinmunización e hipotiroidismo congénito. 3. 3. La isoinmunización anti-D no puede ser descartada en este caso. La ruptura de membranas 5 horas preparto, no parece un factor de riesgo 4. 4. infeccioso relevante, a la hora de considerar sepsis como la causa de la ictericia en este caso. Gráfico de respuestas 1.

1.

Comentario En los casos clínicos de ictericia neonatal es importante identificar posibles factores de riesgo: - Grupo sanguíneo materno (ABO y Rh). Posible isoinmunización anti A/B si madre O y recién nacido A, B o AB. Puede darse en el primer embarazo. Posible isoinmunización anti- Rh si madre Rh – y recién nacido Rh + (90% son anti- D, 10% anti C o E). - Considerar datos de hemorragia/hematomas/fracturas que al reabsorberse elevan la bilirrubina. - Detectar factores de riesgo de infección bacteriana (todos los de sepsis neonatal: corioamnionitis, fiebre materna, cultivo recto- vaginal positivo, IVU en el tercer trimestre, prematuridad… Para la ruptura de membranas se considera >18 horas en RNT. - Síndrome de Arias. Causado por lactancia materna. Es característico que al suspender la lactancia materna, disminuya rápidamente el nivel de bilirrubina, pero habrá que recordar que NO contraindica su administración. - El hipotiroidismo congénito es causa de hiperbilirrubinemia tardía. - La ictericia nunca será fisiológica en las primeras 24 horas de vida. (R/2)

281. En una sospecha de hematoma retroperitoneal la prueba diagnóstica de elección es: 1. Ecografía. 2. TAC. 3. Arteriografía. 4. Lavado peritoneal diagnóstico. Gráfico de respuestas

1. 2. 3. 4.

! !

! Comentario Es una pregunta muy directa. La causa más frecuente de hematoma retroperitoneal son las fracturas pélvicas. Por ello todo politraumatizado debe someterse a una radiografía de pelvis. Debe sospecharse en todo traumatismo con choque hipovolémico sin localización evidente de la hemorragia. Con frecuencia no se acompaña de signos de irritación peritoneal. La prueba de elección para el diagnóstico es el TC. En cuanto al tratamiento siempre debe realizarse exploración quirúrgica en el caso de un trauma penetrante, siendo en el trauma cerrado variable según la localización de las lesiones: si el hematoma es central- superior es siempre quirúrgico (suele lesionarse la VCI), si es en flanco suelen lesionarse los riñones cuyo manejo suele ser conservador, y el hematoma en cavidad pélvica suele presentar lesiones de arterias ilíacas internas y no deben explorarse por riesgo de sangrado masivo. (R/2)

282. Le avisan de de un registro cardiotocográfico patológico, en una gestante de 42 semanas que está de parto. Al romper la bolsa, detecta un líquido amniótico intensamente teñido de meconio. El RN nace con un Apgar al minuto y a los cinco minutos de 4 y 7, precisando aspiración de secreciones y ventilación con mascarilla con presión positiva intermitente. A las dos horas de vida el niño presenta dificultad respiratoria con quejido espiratorio audible sin estetoscopio, disociación tóraco-abdominal ligera, retracción xifoidea y tiraje intercostal marcados, sin aleteo nasal. En la gasometría muestra hipoxemia con hipercapnia. ¿Cuál de las siguientes es CORRECTA? 1. El paciente tiene un score de Silverman de 5. En la radiografía de tórax es frecuente hallar derrame en las cisuras y 2. 2. broncograma aéreo. Otro hallazgo radiológico típico es la hipoventilación, con menos de 7 espacios 3. 3. intercostales en la radiografía AP de tórax. 4. 4. Algunos pacientes se benefician del tratamiento con surfactante intratraqueal. Gráfico de respuestas 1.

Comentario El cuadro descrito corresponde a un síndrome de aspiración meconial. Es característico de RN a término o postérmino, con datos de sufrimiento fetal agudo, Apgar bajo al nacer y líquido amniótico teñido de meconio (el RN hijo de madre diabética es propenso a padecer enfermedad de membrana hialina). La clínica consiste en dificultad respiratoria severa (el Silverman del paciente es de 7), con abombamiento torácico por atrapamiento aéreo (y, por tanto, con signos radiológicos de hiperinsuflación), hipoxemia e hipercapnia. En la radiografía es frecuente encontrar hiperinsuflación, infiltrados algodonosos, atelectasias y aire extraalveolar (el líquido en cisuras es un dato sugestivo de taquipnea transitoria del RN y el broncograma aéreo de enfermedad de membrana hialina). El tratamiento consiste en asistencia respiratoria, antibioterapia y, en ocasiones, surfactante para mejorar la mecánica pulmonar. (R/4)

283. ¿Qué fármaco elegiría para tratar una neumonía en un niño de 5 años de edad, que se acompaña de cefalea, malestar general, tos llamativa, radiografía con patrón intersticial y que en los análisis de sangre presenta trombocitopenia y crioaglutininas?: 1. Eritromicina. 2. Penicilina G sódica. 3. Amoxicilina. 4. Trimetroprim-Sulfametoxazol. Gráfico de respuestas

1. 2. 3. 4.

Comentario ! !

! Pregunta concreta que necesita de buen olfato clínico. Estamos ante una neumonía por Mycoplasma pneumonae siendo el tratamiento de elección en este caso la eritromicina. (R/1)

284. ¿Cuál de los siguientes antiarrítmicos es más nocivo para la glándula tiroides?: 1. Verapamil. 2. Amiodarona. 3. Propranolol. 4. Lidocaína. Gráfico de respuestas

1. 2. 3. 4.

Comentario Esta pregunta puede resultarnos difícil si consideramos que es prácticamente imposible recordar todos los efectos secundarios de cada fármaco, sin embargo hay que prestar especial atención y conocer ciertos efectos adversos característicos de determinados fármacos como en el caso de los antiarrítmicos. La amiodarona, es un antiarrítmico del grupo III muy eficaz en la mayoría de las arritmias tanto supraventriculares como ventriculares malignas, siendo además el fármaco de elección en el contexto de la reanimación cardiopulmonar y en la prevención de los episodios de fibrilación auricular. Pero hay que tener en cuenta la gran cantidad de efectos secundarios, algunos de ellos graves, que puede provocar, como son fibrosis pulmonar, alteraciones de la función tiroidea (respuesta 3 correcta), depósitos corneales, hepatopatía, disminución del crecimiento, alteraciones cutáneas, etc. Además la amiodarona también aumenta los niveles de digoxina pudiendo desencadenar una intoxicación digitálica.(R/2)

285. Nos encontramos ante un hombre de 50 años que presenta febrícula, síndrome constitucional y epistaxis de repetición con dolor en el tabique nasal. Se practica una radiografía de tórax que muestra la presencia de infiltrados pulmonares cavitados bilaterales. Los anticuerpos anti-citoplasma de neutrófilo resultan positivos. Señale cuál de las siguientes afirmaciones en relación a la patología que sospecha NO es cierta: La afectación renal aparece hasta en ¾ partes de los pacientes a lo largo de la enfermedad. Los títulos de c-ANCA se correlacionan positivamente con la actividad de la 2. 2. enfermedad. El tratamiento de elección se basa en ciclofosfamida, a la que se añaden 3. 3. temporalmente corticoides a dosis altas. La afectación más precoz y frecuente se produce a nivel de la vía respiratoria 4. 4. superior. Gráfico de respuestas 1.

1.

Comentario Se trata de una pregunta asequible. Recuerde las características de la granulomatosis de Wegener: vasculitis de mediano y pequeño calibre. Tríada clínica: afectación de la vía respiratoria superior (lo más frecuente y precoz), infiltrados pulmonares bilaterales, cavitados y no migratorios, glomerulonefritis (habitualemente no presente al diagnóstico pero aparece en el 75% en la evolución de la enfermedad). A pesar de ser altamente sensibles (88%) y específicos (95%), los niveles de c-ANCA no se correlacionana con la actividad de la enfermedad (opción 2 falsa).

! !

! El tratamiento se basa en ciclofosfamida 2mg/kg/dia acompañada inicialmente de corticoides a dosis altas (1mg/kg/dia) con descenso durante los primeros meses.(R/2)

286. Mujer de 50 años en cuya mastografía anual se visualizan en el cuadrante infero ínterno de la mama izquierda un grupo de 7 microcalcificaciones de tamaño simétrico agrupadas anárquicamente. En la exploración clínica no se detectan nódulos palpables. ¿Cuál sería su conducta a seguir? 1. Repetir la mastografía en 6 meses para ver si el hallazgo sigue estable. 2. Realizaría una biopsia con aguja gruesa en consulta. 3. Realizaría una biopsia dirigida con arpón. Tranquilizaría a la paciente, porque ese tipo de microcalcificaciones representan 4. 4. un hallazgo muy común propio del envejecimiento del tejido mamario. Gráfico de respuestas 1. 2. 3.

Comentario La aparición de microcalcificaciones agrupadas anárquicamente en número igual o superior a 5 de tamaño simétrico constituye el signo sospechoso de malignidad que aparece más precozmente en la mamografía. Si no se corresponden a una lesión palpable no podremos realizar BAG o PAAF en consulta por lo que habrá que recurrir al marcaje de la lesión con un arpón y a su biopsia diferida.(R/3)

287. En un paciente portador de una prótesis valvular aórtica que va a realizarse una endodoncia y es alérgico a penicilina, estarían indicados los siguientes fármacos como profilaxis de endocarditis, EXCEPTO: 1. Vancomicina. 2. Gentamicina. 3. Teicoplanina. 4. Azitromicina. Gráfico de respuestas

1. 2. 3. 4.

Comentario Es importante tener unas nociones básicas de la cobertura que tiene cada familia de antibióticos, con ello se puede responder a unas cuantas preguntas, entre ellas ésta. En los procedimientos dentales, la profilaxis buscará cubrir una posible bacteriemia por S. viridans, microorganismo más frecuentemente implicado. De los antibióticos que hay entre las respuestas la gentamicina es el único que sólo cubre gram negativos, por lo que sería la respuesta falsa. Vancomicina y teicoplanina cubren grampositivos, incluso el S. epidermidis o el S. aureus oxacilin resistente. La azitromicina es un macrólido que puede cubrir también la legionella, la Bordetella pertussis (tos ferina) y neumonía atípica.(R/2)

288. ¿Cuál de las siguientes afirmaciones es FALSA?: 1. 2.

! !

La "artritis colítica" periférica sigue una evolución paralela a los brotes de la enfermedad intestinal. Los pacientes con enfermedad intestinal inflamatoria crónica pueden presentar 2. artritis axial indistinguible de la espondiloartritis idiopática. 1.

! Un 40% de los casos de espondilitis asociada a enfermedad intestinal son mujeres. Los brotes de la espondilitis asociada a la enfermedad intestinal inflamatoria 4. 4. crónica corren paralelos a los de dicha enfermedad. Gráfico de respuestas 3.

3.

Comentario La artritis de articulaciones periféricas asociada a la enfermedad inflamatoria intestinal tiene característucamente un curso paralelo a los brotes intestinales. Es más probable que se produzaca en los pacientes que tienen complicaciones como abcesos, poliposis pseudomembranosa, lesiones perianales, eritema nodoso, estomatitis, uveitis o pioderma gangrenoso. No muestra predominio sexual. La afectación del esqueleto axial tiene un curso independiente de los síntomas gastrointestinales y predomina en hombres.(R/4)

289. La mayor frecuencia de muerte temprana en pacientes con lesión pancreática se debe a: 1. Absceso pancreático. 2. Fistula pancreática. 3. Fistula duodenal. 4. Lesión vascular asociada. Gráfico de respuestas

1. 2. 3. 4.

Comentario Al estar el páncreas situado en el retroperitoneo, es muy frecuente que una lesión pancreática se asocie a lesión de los grandes vasos (aorta o vena cava) que también se situan en este espacio anatómico. Respuesta 4 correcta.

290. Mujer de 34 años, que acude al servicio de urgencias con dolor en cuello de 3 días de evolución, febrícula y nerviosismo, diagnosticada de amigdalitis aguda por médico de atención primaria y tratada con ibuprofeno y amoxicilina-clavulánico. El dolor persiste y aparecen palpitaciones e insomnio, motivo por el que acude a urgencias alarmada por la mala evolución del cuadro. En los exámenes de laboratorios realizado se detecta una VSG de 85. La paciente

! !

! fue dada de alta con el mismo tratamiento y remitida a consultas de endocrinología, a la que acude con el resultado de la gammagrafía tiroidea solicitada (ver imagen). ¿Cuál es el diagnóstico más probable y el tratamiento que Ud. pautaría? Ademona folicular hiperfuncionante - metimazol y betabloqueantes, previo a radioyodo. Tiroditis subaguda de De Quervain - betabloqueantes y corticoides dada la poca 2. respuesta a AINEs. 3. 3. Struma ovari - antitiroideos y cirugía. 4. 4. Bocio multinodular tóxico - antitiroideos y radioyodo. Gráfico de respuestas 1. 2.

1.

Comentario La paciente padece una tiroiditis subaguda, vírica o de De Quervain. Posiblemente, el diagnóstico de amigdalitis aguda que emitió el médico de atención primaria fuese correcto, pero de origen vírico, afectando al tiroides secundariamente. Es lo que sucede habitualmente en esta entidad: infección vírica unos días antes, normalmente de vías altas, afectándose después el tiroides. Desde el punto de vista clínico, destacan los síntomas de hipertiroidismo junto con dolor en región anterior del cuello. Encontraríamos una TSH suprimida y la VSG habitualmente elevada. En cuando a la gammagrafía acompañante, como sucede en la práctica totalidad de las tiroiditis, vemos abolida la captación de la glándula.(R/2)

291. When a patient has a parathyroid hyperplasia, pancreatic tumors and pituitary tumors, which is the diagnosis? 1. Cowden's syndrome. 2. MEN type IIA. 3. Von Recklinghousen disease. 4. MEN type I. Gráfico de respuestas

1. 2. 3. 4.

Comentario MEN type I. Wermer's syndrome or MEN type I is a disorder defined by parathyroid hyperplasia, pancreatic tumors and pituitary tumors. Hypercalcemia is not a usual finding in a Zollinger-Ellison syndrome and when it appears it suggest a multiple endocrin disease and a worse prognosis.(R/4)

292. A 13-year-old child is brought to the pediatrician's office for a routine check-up. He underwent an abdominal intervention one year ago. Findings in peripheral blood smear are described as: "occasional red blood cells with bluish nuclear remnants" are often seen in one of the following processes: 1. Splenectomy. 2. Hemoglobin precipitation. 3. Pure red blood cells aplasia. 4. Parvovirus infection. Gráfico de respuestas

1. 2. 3. 4.

Comentario Splenectomy. The red cells described in this blood smear have Howell–Jolly bodies inclusions. These are histopathological findings of basophilic nuclear remnants (clusters of DNA) in circulating erythrocytes and are seen with markedly decreased splenic function.(R/1)

! !

!

293. Respecto a las siguientes afirmaciones sobre la infección en el paciente neutropénico, señale la INCORRECTA: El tratamiento empírico inicial con una única cefalosporina antipseudomonas puede ser adecuado. 2. El ectima gangrenoso está causado de forma exclusiva por Pseudomonas. 2. 3. 3. El síndrome de Sweet puede ser un cuadro paraneoplásico. 4. 4. La tiflitis cursa clínicamente como una apendicitis. Gráfico de respuestas 1.

1.

Comentario Aunque el ectima gangrenoso es una infección típicamente asociada a P. aeruginosa en el paciente neutropénico, también se ha descrito en el contexto de otros gramnegativos (respuesta 2 falsa).

294. Siendo la sospecha clínica la de apendicitis aguda, ¿cuál sería su actitud terapéutica?: 1. Sueroterapia y analgésicos. 2. Intervención quirúrgica urgente: apendicectomía. 3. Observación domiciliaria. 4. Laparotomía exploradora. Gráfico de respuestas

1. 2. 3. 4.

Comentario Lo más probable es que se pregunte la apendicitis en forma de caso clínico, no olvide revisar los criterios de Alvarado. El tratamiento de una apendicitis aguda siempre es quirúrgico (apendicectomía) puesto que dejada a su evolución natural puede desembocar en una perforación o una peritonitis difusa. El tratamiento se completa con antibioticoterapia profiláctica. Sólo en el caso de que el diagnóstico se haga en la fase de plastrón apendicular se puede diferir la cirugía tras un par de semanas de tratamiento antibiótico intravenoso.(R/2)

295. Una mujer joven acude a nuestra consulta refiriendo astenia, pérdida de peso e hiperpigmentación progresiva. El estudio funcional de la paciente muestra un cortisol plasmático en límites bajos y ausencia de respuesta del cortisol sérico al estímulo de ACTH. ¿Cuál será probablemente la causa de su insuficiencia suprarrenal?: 1. Tuberculosis. 2. SIDA. 3. Amiloidosis. 4. Adrenalitis autoinmune. Gráfico de respuestas

1. 2. 3. 4.

Comentario Aunque esta pregunta se formule en forma de caso clínico, hace referencia a la etiología más frecuente de una patología que puede deducirse por los datos aportados. Ante una paciente joven, que presente astenia intensa, pérdida de peso, y en especial, hiperpigmentación lo primero que hay que descartar es la existencia de una insuficiencia suprarrenal primaria (Enfermedad de Addison). Dentro de las causas de esta enfermedad, la más frecuente es la adrenalitis autoinmune,

! !

! siendo la segunda en frecuencia la tuberculosis. Otras causas menos frecuentes serían la existencia de infecciones oportunistas en pacientes con SIDA, las metástasis y patologías como la amiloidosis.(R/4)

296. En gerontopsiquiatría, la melancolía involutiva se caracteriza por todos, MENOS uno de los siguientes aspectos: 1. Tendencia a la autodepreciación y autoacusación. 2. Ideas de culpabilidad. 3. Trastornos confusionales y oníricos. 4. Disminución de la autoestima. Gráfico de respuestas

1. 2. 3. 4.

Comentario La "melancolía involutiva" ha desaparecido de las clasificaciones porque no es más que una depresión endógena severa ("melancolía") de la ancianidad. Ahora bien, tiene sus peculiaridades debido al momento del ciclo vital y a los factores orgánicos: predominan las preocupaciones hipocondríacas (que a veces alcanzan un rango delirante de severidad, como en el Síndrome de Cottard), y se asocia más que en la población general con degeneración cerebral vascular (no es infrecuente que años después de una melancolía involutiva aparezca una demencia vascular). Lo que claramente excluye el diagnóstico de demencia y de depresión, al menos provisionalmente hasta que se resuelva, es la clínica confusional- oniroide (es decir, la disminución del nivel de conciencia tipo delirium).(R/3)

297. Niño de 5 años de edad que, estando previamente bien, despierta por la noche con fiebre de 39.8ºC, voz apagada, intenso babeo, estridor inspiratorio con escasa tos. A la exploración destaca mal estado general, estridor inspiratorio e intenso tiraje supraesternal y subcostal, así como tendencia a la hiperextensión del cuello. Se realiza radiografía lateral de faringe, observando ocupación de espacio supraglótico. En la biometría hemática destaca una leucocitosis con desviación izquierda. ¿Qué medida terapéutica NO estaría aconsejada en este caso?: 1. Aplicar oxígeno e intentar obtener una vía aérea artificial en quirófano. 2. Aplicar adrenalina racémica y corticoides. 3. Usar las medidas generales para descender la hipertermia que sufre el paciente. 4. Mantener tranquilo al paciente. Gráfico de respuestas

1. 2. 3. 4.

Comentario Pregunta importatísima para el ENARM, es un cuadro de epiglotitis de libro. Estamos ante un cuadro de epiglotitis aguda de tal forma que la 1, 3 y 4 serían opciones terapeúticas a utilizar en este cuadro. Sin emabrgo, la número 2 se utiliza en el tratamiento de las laringitis agudas (estridulosas) o laringotraqueitis viral. Recuerde que por nada del mundo debes interar explorar la vía aerea porque sería peor, debe asegurarla.(R/2)

298. ¿Cuál de las siguientes NO puede considerarse una reacción de hipersensibilidad retardada?:

! !

! 1. Neumonitis en el cuidador de aves. 2. Histoplasmosis. 3. Beriliosis. 4. Linfogranuloma venéreo. Gráfico de respuestas

1. 2. 3. 4.

Comentario Las reacciones de hipersensibilidad es un tema importante dentro de la inmunología. Las reacciones de hipersensibilidad (RHS) se clasifican en 4 tipos, cada uno con sus características y lo que es realmente difícil es clasificar las distintas patologías inmunológicas con el grupo al que pertenece. La reacción tipo IV o retardada es mediada por células y necesita una sensibilización previa. El ejemplo típico es el Mantoux (PPD), pero encontramos otros como el rechazo agudo de trasplantes, los granulomas y la hipersensibilidad por contacto. La neumonitis del cuidador de aves (neumonitis por hipersensibilidad) se produce por la mezcla de RHS tipo III y IV, lo mismo que ocurre en la histoplasmosis y la beriliosis, por lo que muy frecuentemente encontramos granulomas en la anatomía patológica. La Aspergilosis broncopulmonar alérgica es un trastorno complejo en el que se han visto mezcladas varias RHS: tipo I y III fundamentalmente, aunque también tipo IV. El linfogranuloma venéreo es una ETS provocada por Clamydia trachomatis serotipos L1, L2 y L3, por ello no influye ninguna RHS.(R/4)

299. Todas las formas de presentación de la patología pancreática, son frecuentes, EXCEPTO: 1. Encefalopatía pancreática. 2. Dolor de espalda. 3. Síndrome tóxico-constitucional. 4. Síntomas neuropsiquiátricos. Gráfico de respuestas

1. 2. 3. 4.

Comentario La encefalopatía pancreática en una entidad que se asocia a las pancreatitis agudas y que aparece con muy baja frecuencia. Se puede apreciar clínica compatible con encefalopatía en el 4 a 20% de todos los pacientes con pancreatitis aguda. Los síntomas pueden incluir desorientación, confusión, delirio, y alucinaciones. Debido a que el alcohol y el síndrome de deprivación también producen estos síntomas a veces es imposible discernir cual es la causa subyacente. El edema cerebral, las hemorragias y la necrosis focal, presumiblemente secundarias a alteraciones del flujo sanguíneo microvascular han sido relacionadas con episodios de pancreatitis aguda.(R/1)

300. La oxigenoterapia crónica domiciliaria está indicada en el tratamiento de la EPOC cuando: El paciente cumple el tratamiento broncodilatador completo y la PaO2 es menor de 65 mmHg. El paciente cumple el tratamiento broncodilatador completo y la PaO2 es inferior 2. 2. a 55 mmHg en las agudizaciones. 3. La disnea es de mínimos esfuerzos. 3. El paciente cumple el tratamiento broncodilatador completo y la PaO2 es inferior 4. 4. a 55 mmHg, estando el paciente en situación basal. Gráfico de respuestas 1.

1.

Comentario ! !

! Pregunta sencilla que no debe fallar en ningún momento. Es importante saber que las únicas dos medidas que mejoran la supervivencia en el paciente EPOC son el dejar de fumar y la oxigenoterapia domiciliaria si está indicada. Esta indicación es en pacientes que no fumen y que en tratamiento broncodilatador completo su pO2 basal es menor que 55mmHg y si está entre 55- 60 en caso de presentar alguna complicación como hipertensión pulmonar, cor pulmonale, ICC, arritmias, policitemia o reducción del intelecto. Recuerde que el uso es de16h/día al menos y no se realiza descanso nocturno.(R/4)

301. Un paciente de 27 años es incapaz de impedir el desplazamiento anterior del fémur sobre la tibia cuando la rodilla está flexionada. ¿Cuál de los siguientes ligamentos está dañado?: 1. Cruzado anterior. 2. Colateral peroneo. 3. Rotuliano. 4. Cruzado posterior. Gráfico de respuestas

1. 2. 3. 4.

Comentario La maniobra de exploración que nos describen es la “prueba del cajón posterior”. El desplazamiento hacia anterior del fémur sobre la tibia (o al revés), con la rodilla flexionada a unos 90º, nos indica la posible rotura del ligamento cruzado posterior. Este ligamento suele lesionarse en traumatismos directos sobre la tibia con la rodilla en flexión. Además, en casos crónicos puede observarse una deformidad en recurvatum – la rodilla se hiperextiende al no existir el tope del ligamento cruzado posterior. La lesión aislada del ligamento cruzado posterior puede ser subsidiaria de tratamiento conservador (potenciar cuádriceps), pero si se combina con la lesión de una o más estructuras laterales (inestabilidad posterolateral) es necesaria la reconstrucción quirúrgica.(R/4)

302. Respecto a la prueba de la tuberculina, todas las afirmaciones siguientes son correctas, EXCEPTO: La lectura de la reacción se debe hacer midiendo el diámetro de eritema medido a las 48-72 horas. La reacción positiva del PPD sólo indica que probablemente hay alguna infección 2. 2. micobacteriana. La repetición de las cutirreacciones puede hacer que aumenten de tamaño (efecto 3. booster), aunque no pueden producir una reacción positiva en un sujeto no 3. infectado. 15% de los sujetos con tuberculosis pulmonar activa reciente y 50% de los 4. 4. enfermos con tuberculosis miliar presentan un Mantoux negativo. Gráfico de respuestas 1.

1.

Comentario Sobre la técnica de realización del Mantoux, - Lo que se mide es la INDURACIÓN, y no el eritema. - El inóculo es a nivel INTRADÉRMICO, no subcutáneo.

! !

! Y, efectivamente, lo que significa un Mantoux positivo es que existe INFECCIÓN, y no ENFERMEDAD tuberculosa.(R/1)

303. María es una mujer de 29 años de edad, embarazada de 28 semanas, que debuta con un cuadro clínico de dolores musculares generalizados, con escalofríos y sensación distérmica, sin foco aparente. Tras 10 días de evolución, se añade al cuadro clínico un dolor a nivel de la faringe, con cuadro febril de hasta 38.8ºC, cefalea y adenopatías cervicales. Se diagnostica de un cuadro mononucleósico, con germen identificado como citomegalovirus. Respecto a la infección congénita por CMV, son todas correctas, EXCEPTO: 1. La mayoría de los infectados intraútero están asintomáticos al nacimiento. La microcefalia, calcificaciones periventriculares y la coriorretinitis son los 2. 2. hallazgos más característicos de la forma sintomática. La IgG positiva anti-CMV en el suero del recién nacido es diagnóstica del 3. 3. proceso. 4. 4. Es causa frecuente de sordera y retraso mental. Gráfico de respuestas 1.

Comentario De todas las infecciones connatales, la debida al citomegalovirus es la más frecuente. Se da especialmente cuando la madre padece la primoinfección o cuando está coinfectada por CMV y por VIH. - Si la madre adquiere la infección durante la primera mitad del embarazo, su hijo puede presentar la tríada sintomática (situación poco frecuente): microcefalia, calcificaciones periventriculares y corioamnionitis. - Si la adquiere en la segunda mitad (más frecuente), estará asintomático al nacer y después desarrollará hipoacusia neurosensorial. Para diagnosticarlo, puede tomar un cultivo de orina en la primera semana de vida o determinar la IgM antiCMV en el recién nacido. Lógicamente, la IgG carece de valor, puesto que esta inmunoglobulina atraviesa la placenta y, por ello, es posible encontrarla en sangre del neonato sin que esté infectado.(R/3)

304. Mujer de 46 años que en una rectosigmoidoscopia, prescrita por estreñimiento y dolor abdominal de reciente aparición, se detecta una masa ulcerada en cara anterior de sigma. Dos años antes había sido intervenida de un carcinoma de endometrio y en su familia existían antecedentes de varios familiares fallecidos por cáncer intestinal. A la exploración física se aprecian lesiones cutáneas en brazos y tronco sugestivas de tumores sebáceos. El diagnóstico más probable en este caso sería el de síndrome de: 1. Peutz-Jeghers. 2. Cowden. 3. Muir-Torre. 4. Turcot. Gráfico de respuestas

1. 2. 3. 4.

Comentario

! !

! Una pregunta muy peligrosa. La historia familiar de cáncer de colon, teniendo también en cuenta el cáncer de endometrio, es muy sugestiva de síndrome de Lynch tipo II… Pero, si nos dan algún dato suele ser por algo. Si añade a esto los tumores sebáceos, debe responder inmediatamente el síndrome de Muir-Torre. De hecho, este síndrome está muy relacionado con el Lynch II, incluso desde el punto de vista genético, por lo que los hallazgos descritos también podrían encontrarse en él. Sin embargo, la presencia de tumores sebáceos sólo pueden encontrarse en la respuesta 3.

305. ¿El limite recomendado de 15 mmHg para el neumoperitoneo, tiene relación con que parámetro? 1. Distensibilidad de la pared abdominal. 2. Compresibilidad de la pared arterial. 3. Presión arterial media. 4. Presión venosa. Gráfico de respuestas

1. 2. 3. 4.

Comentario El límite de 15mmHg de presión de aire en u pneumoperitoneo es el limite a partir del cual se impediria el retorno venoso. A partir de 15 mmHg se colapsarian las venas abdominales y no podria retornar la sangre al corazón.(R/4)

306. La enfermedad de Köhler es una osteocondrosis que afecta, ¿qué hueso? 1. Tuberosidad anterior de la tibia. 2. Polo inferior de la rótula. 3. Tuberosidad posterior del calcáneo. 4. Escafoides tarsiano. Gráfico de respuestas

1. 2. 3. 4.

Comentario Las osteocondrosis son enfermedades poco importantes en el ENARM. Sin embargo, si tiene tiempo, no estaría de más repasar los nombres y la localización típica de cada una (sería una pregunta difícil y valiosa, porque no la acertaría más de un 10-15%). Las distintas osteocondrosis se fueron describiendo con el nacimiento de la radiología, por ello cada una lleva un epónimo distinto. La osteocondrosis de Köhler se localiza en el escafoides tarsiano. Tenga cuidado, porque esta de Köhler (I) se puede confundir con la de Köhler II o enfermedad de Freiberg, que se localiza en la cabeza de los metatarsianos.(R/4)

307. ¿Cuál es la conducta a adoptar en un recién nacido cuya madre se le diagnosticó varicela durante el parto? 1. Aplicar la vacuna de la varicela dentro de las 24 horas del nacimiento. Aplicar la gammaglobulina específica hiperinmune dentro de las 72 horas del 2. 2. nacimiento. 3. Indicar aciclovir a partir de los 7 días del nacimiento. 3. 4. Conducta expectante, medidas de aislamiento de contactos hasta el alta. 4. Gráfico de respuestas 1.

Comentario La profilaxis postexposición en RN con madres con varicela es la siguiente:

! !

! Inmunoglobulina inmune contra varicela-zoster, la cual previene la transmisión de varicela en los neonatos expuestos, disminuye la sintomatología. La CDC recomienda la administración de la Ig a los RN con exposición reciente a VVZ en casos de: neonatos cuyas madres presentan signos y síntomas de varicela al rededor del parto (5 días antes y 2 días después). Respuesta 2 correcta.

308. Respecto a la hipertrofia benigna de próstata es FALSO que: 1. Es la principal causa de obstrucción del tracto urinario inferior en el hombre. 2. Puede condicionar insuficiencia renal crónica. 3. Puede originar litiasis a nivel vesical. La intensidad de los síntomas es directamente proporcional al tamaño de la 4. 4. glándula prostática. Gráfico de respuestas 1. 2. 3.

Comentario La hiperplasia benigna de próstata es uno de los temas más preguntados en esta asignatura. Esta pregunta, en concreto, es tan sencilla que la falsedad de la opción 4 debería reconocerse casi al instante. En la hiperplasia benigna de próstata no existe una correlación entre el tamaño de la glándula y la presencia de síntomas (respuesta 4 falsa). Existen próstatas pequeñas pero muy obstructivas donde la hiperplasia afecta de forma preferente a la región periuretral, lo que produce una clínica desproporcionada si se compara con el volumen de la glándula. Por el contrario, existen pacientes con próstatas mucho más voluminosas, pero con escasa afectación periuretral, y por tanto, con menos síntomas obstructivos de los que cabría esperar si sólo tuviéramos en cuenta el tamaño prostático. El resto de las opciones son claramente correctas: efectivamente, es la causa más frecuente de obstrucción urinaria en el varón (respuesta 1 co-rrecta). En cuanto a las respuestas 2 y 3, son claramente correctas. Aparte, en el ENARM, cuando existen dudas, resulta imprudente desafiar al poder del “puede”…(R/4)

309. Masculino de 76 años con antecedentes de HTA en tratamiento crónico con tiacidas. Acude a urgencias porque la familia dice que le encuentra "raro". En el control de laboratorio obtenido se detecta Na de 123 mmol/l, K 4.5 mmol/l, Cr 1 mg/dl, Urea 40 mg/dl. En la exploración física presenta signos de depleción de volumen. ¿Cuál es la causa más probable de la hiponatremia que presenta?: 1. Hiponatremia por diuréticos. 2. SIADH. 3. Cirrosis. 4. Síndrome pierde sal. Gráfico de respuestas

1. 2. 3. 4.

Comentario Pregunta típica de diagnóstico diferencial de hiponatremia. Para este tipo de preguntas es importante buscar los antecedentes del paciente y la situación hídrica. En este caso, el paciente está deshidratado y ha estado tomando tiacidas, y además es mayor, hecho que asocia

! !

! fuertemente a efectos secundarios de tiacidas. El otro cuadro compatible con la descripción descrita es el sídrome pierde sal cerebral, que se caracteriza por elevada natriuresis y deshidratación en el contexto habitualmente de un EVC, factor que no presenta en este caso.(R/1)

310. ¿Cuál de los siguientes antiarrítmicos está más indicado en el tratamiento de una fibrilación auricular secundaria a un síndrome de Wolff-Parkinson-White? 1. Amiodarona. 2. Flecainida. 3. Propranolol. 4. Verapamil. Gráfico de respuestas

1. 2. 3. 4.

Comentario En esta pregunta la RESPUESTA CORRECTA es la número 2 (flecainida). La fibrilación auricular, en el contexto del Wolff-Parkinson-White, es una urgencia cardiológica. En una persona sana, los impulsos originados en la fibrilación auricular (unas 600 ondas por minuto), afortunadamente, son filtrados por el nodo AV. Éste sólo deja pasar algunos, por lo que la respuesta ventricular es característicamente irregular, en general entre 100 y 150 lpm (esto es muy variable, dependiendo de la capacidad de conducción del nodo AV en cada individuo). En un paciente con vía accesoria, la situación es muy distinta. Es posible que ésta conduzca de forma anterógrada (de aurícula a ventrículo), con períodos refractarios cortos, es decir, con buena capacidad de conducción. La vía accesoria no se comporta como un “filtro” de impulsos, como haría el nodo AV… Por ello, por esta vía podrían pasar todos o casi todos los impulsos auriculares al ventrículo. En consecuencia, si la aurícula va a 600, el ventrículo recibiría 600 impulsos eléctricos por minuto… Y eso puede inducir a fibrilación ventricular. Cuando se produce esta situación, se habla de fibrilación auricular preexcitada. En estos casos, se precisa del empleo de fármacos que aumenten la refractariedad de las células de la vía accesoria, siendo de elección la procainamida, flecainida o propafenona. Nunca deben emplearse fármacos que bloqueen el nodo AV, pues esto puede facilitar la conducción por la vía accesoria de forma indirecta, por lo que están contraindicados digoxina, verapamil o diltiacem. Los betabloqueantes frenan el nodo AV y podrían también frenar la vía, pero el efecto es muy variable y, en general, no se recomiendan. La amiodarona intravenosa, aparte de su efecto antiarrítmico, tiene un efecto calcioantagonista marcado. Como consecuencia de este efecto, induciría una vasodilatación periférica, cuya respuesta compensatoria sería una estimulación simpática refleja. Las catecolaminas serían contraproducentes en esta circunstancia, porque acortan el período refractario de las vías de conducción (entre ellas, la vía accesoria), con lo que llegarían más impulsos al ventrículo, pudiendo desencadenarse una fibrilación ventricular.(R/2)

311. Un paciente de 62 años, ex fumador reciente, consulta por disnea de varios meses de evolución y tos no productiva. En ambas manos existen acropaquias muy evidentes. En la auscultación se detectan crepitantes teleinspiratorios y en la Rx tórax se aprecia un patrón reticular. La pletismografía objetiva una pérdida de volumen y la TC confirma los hallazgos radiográficos. Si se realiza un lavado broncoalveolar, sería esperable:

! !

! 1. Predominio de los linfocitos CD8+ sobre los CD4+. 2. Aumento de los neutrófilos. 3. Aumento de los eosinófilos por encima del 50%. 4. Incremento de las células de Langerhans. Gráfico de respuestas

1. 2. 3. 4.

Comentario Caso clínico típico. Los datos que te deben llevar a pensar que se trata de un paciente con fibrosis pulmonar idiopática son: 60 años, disnea y tos seca, crepitantes teleinspiratorios, acropaquias y patrón reticular. En el LBA típico hay aumento de neutrófilos.(R/2)

312. ¿Cuál de las siguientes característica aparece con más frecuencia en la EPOC tipo bronquitis que en la tipo enfisema?: 1. Disnea. 2. Tos y expectoración abundante. 3. Disminución del murmullo vesicular en la auscultación pulmonar. 4. Alteración de la capacidad de difusión. Gráfico de respuestas

1. 2. 3. 4.

Comentario Pregunta sencilla para la cual debe tener claras aquellas cosas que en un caso clínico nos aproximan más a enfisema o más a bronquitis crónica, ya que este tema es importante. Debe saber identificar conceptos que nos indiquen bronquitis crónica como tos y expectoración abundantes, infecciones y agudizaciones frecuentes, hábito pícnico, cianosis, alteraciones gasométricas graves, poliglobulia, HTP, cor pulmonale y escasa alteración en la DLCO. Por el contrario si lo que quieren es que enfoquemos el caso a un enfisema encontraremos expresiones como la disnea intensa, hábito asténico, poca alteración gasométrica, alteraciones de la DLCO, retracción elástica alterada, intenso esfuerzo respiratorio, disminución del murmullo vesicular y RX característica con hiperinsuflación y silueta cardiaca alargada. En nuestro caso nos preguntan algo muy sencillo, ya que es la tos y expectoración abundante lo que indica bronquitis crónica, y además es lo que la define clínicamente.(R/2)

313. A 60-year-old female is diagnosed with multifocal invasive ductal breast carcinoma and is scheduled to undergo mastectomy plus axillary dissection and postoperative chemotherapy. What is the best timing for breast reconstruction surgery? 1. It depends on the number of positive nodes. 2. Postoperative chemotherapy is a contraindication to immediate reconstruction. Immediately, since breast reconstruction does not interfere with the natural 3. 3. history of the disease. 4. In patients aged 60 or over, it is better to use external prosthesis. 4. Gráfico de respuestas 1. 2.

Comentario Pregunta sencilla sobre el cáncer de mama. La reconstitución mamaria se puede realizar inmediatamente, ya que no está contraindicada su realización a pesar de los tratamientos. Es muy habitual colocar los expansores mamarios inmediatamente posterior a la cirugía. Respuesta correcta 3.

! !

! 314. Aunque la pseudoartrosis es una complicación que puede aparecer en cualquier fractura, su presencia es más frecuente en determinadas localizaciones ¿Cuál de los siguientes huesos NO suele cursar con pseudoartrosis cuando se produce una fractura del mismo? 1. Escafoides. 2. Clavícula. 3. Astrágalo. 4. Tibia. Gráfico de respuestas

1. 2. 3. 4.

Comentario La pseudoartrosis o ausencia de consolidación, en este caso avascular, es una complicación relativamente frecuente en todas las localizaciones reflejadas en las opciones excepto la clavícula. La fractura de clavícula, sobre todo las de 1/3 medio, tienen una tasa de excelentes resultados mediante tratamiento conservador, con muy bajo índice de trastornos de consolidación, Ya que se trata de un hueso bien vascularizado. Se ha visto que las fracturas tratadas inicialmente mediante reducción y osteosíntesis presentan una tasa elevada de pseudoartrosis, de alrededor del 5%.(R/2)

315. Paciente que acude a urgencias con piel seca, mucosas secas, lagrimas disminuidas, llenado capilar de 2 segundos, oliguria, ruidos cardiacos de buena intensidad. ¿Qué tipo de deshidratación tiene? 1. Moderada. 2. Severa. 3. Grave. 4. Leve. Gráfico de respuestas

1. 2. 3. 4.

Comentario Hay que tener presente que los grados de deshidratación son distintos en lactantes, niños y adultos, pues se considera más grave en lactantes y niños. El grado de deshidratación se corresponde con el % de peso perdido pero a veces no se dispone de ese dato y se tiene que recurrir a signos clínicos que de manera indirecta permiten estimar el grado de deshidratación. En pediatria, clasificamos la deshidratación en: - Leve o de primer grado: Pérdida de < 5% (lactantes), < 3% (niños). - Moderada o de segundo grado: Pérdida 5-10% (lactantes), 3-7% (niños) - Grave o de tercer grado: > 10% (niños), > 7 % (lactantes) Si tenemos en cuenta los signos clínicos, la clasificación sería: 1. Deshidratación leve: sin signos ni síntomas.

! !

! 2. Deshidratación moderada: sed, sequedad de piel y mucosas, comportamiento inquieto o irritable, ojos hundidos. 3 .Deshidratación grave: los síntomas se agravan; choque, con pérdida parcial del conocimiento, falta de diuresis, extremidades frías y húmedas, pulso rápido y débil, tensión arterial baja o no detectable, y palidez. En cuanto a los adultos: - Deshidratación leve: La sed es el único síntoma, pero es necesario saber que la sed no es un signo específico del déficit de agua, ya que aparece en otros síndromes (diabetes descompensada, hemorragias o estados de ansiedad) - Deshidratación moderada: piel y mucosas secas, paciente debilitado, con taquicardia, hipertermia leve, la disminución del peso corporal en un 5%. Además disminución de la diuresis y aumento del residuo en orina. - Deshidratación grave: Al cuadro anterior se le agregan graves trastornos de la conciencia (obnubilación, delirio, estupor, que pueden finalizar en un estado de coma). Se profundiza la hipotensión arterial y la hipertermia y la muerte sobreviene cuando la pérdida de agua alcanza entre 6 y 10 litros en una persona de talla adulta. En esta pregunta, independientemente de que el paciente sea un niño o un adulto se trata de un caso de deshidratación moderada. Ten en cuenta que la respuesta 2 y 3 podrian ser el mismo grado de deshidrtación, pues severa y grave son sinónimos.(R/1)

316. Usted sabe que el síndrome de ovario poliquístico (SOP) se caracteriza por una alteración hormonal compleja. Respecto a la fisiopatología, es FALSO que: 1. Afecta al 4-10% de las mujeres en edad reproductiva. Existe una concentración sérica media de LH elevada, con FSH normal o en el 2. 2. límite inferior de la normalidad. En un 20-30% de casos aparece hiperprolactinemia moderada, que alcanza 3. 3. niveles anovulatorios. El DHEA-S se encuentra elevado en el 20% de las pacientes con ritmos 4. 4. circadianos de ACTH y cortisol normales. Gráfico de respuestas 1.

Comentario El síndrome de ovario poliquístico es muy frecuente, afectando al 1-5% de las mujeres, aumentando hasta el 10% en edad reproductiva, incluso más en algunas series. La fisiopatología es compleja y en algunos aspectos no conocida del todo. Aparece un aumento de los pulsos de la GnRH por parte del hipotálamo, el cual estimula un aumento constante de la LH, la cual duplica a la FSH. La prolactina puede elevarse en un 20-30% de las pacientes, pero NO llega a alcanzar niveles anovulatorios. Los andrógenos suprarrenales pueden elevarse en torno al 20% de las pacientes.(R/3)

317. Un niño de 8 años presenta un cuadro de cefalea, fiebre, exantema maculopapular generalizado y dolor centrotorácico punzante. En las exploraciones complementarias se aprecia en el LCR una pleocitosis linfocitaria con glucorraquia y proteinorraquia normales, y en el electrocardiograma se observa una elevación generalizada del

! !

! segmento ST con concavidad hacia arriba. ¿Qué diagnóstico es el más probable ante dicha constelación de hallazgos? 1. Infección por Mycoplasma pneumoniae. 2. Infección por virus herpes simple 2. 3. Infección por Coxsackie B. 4. Infección por el virus de la parotiditis. Gráfico de respuestas

1. 2. 3. 4.

Comentario Una elevación generalizada del ST hacia arriba es sugestivo de una pericarditis, sobre todo si además es cóncavo. Por otra parte, sabe que la causa más frecuente de pericarditis aguda es vírica (Coxsackie B), por lo que debe elegir la respuesta 3. En cuanto al resto del cuadro clínico, no era una pregunta tan difícil. Prácticamente todos los virus pueden justificar exantemas cutáneos, y los enterovirus (entre ellos el coxsackie) son también una causa frecuente de meningitis vírica. El mismo virus, por tanto, puede justificar todo lo que nos cuentan de este niño…

318. Los betabloqueantes son útiles en el tratamiento de la angina de pecho porque, entre otras acciones: 1. Dilatan las arterias coronarias. 2. Disminuyen la frecuencia cardiaca. 3. Aumentan la tensión arterial. 4. Aumentan el poder contráctil del corazón. Gráfico de respuestas

1. 2. 3. 4.

Comentario Es una pregunta muy fácil y todos deben conocer el verdadero motivo por el cual los betabloqueantes ejercen un efecto tan favorable en los pacientes con angina de pecho. Esto es debido a que al disminuir la frecuencia cardiaca, están disminuyendo el trabajo cardiaco, lo que a su vez provoca una disminución de los requerimientos de oxígeno. Este efecto es muy beneficioso en estos pacientes. Los efectos secundarios de los beta- bloqueantes pueden ser bradicardia, bloqueos cardíacos, disminución de la contractilidad cardiaca con insuficiencia cardiaca, vasoconstricción periférica, broncoespasmo, fatiga muscular, depresión, pesadillas, alucinaciones, hiperglucemia, disminución del flujo renal....(R/2)

319. En la glomerulonefritis aguda postestreptocócica, una de las siguientes afirmaciones es FALSA: El título de antiestreptolisina O (ASLO) se eleva en la mayoría de los casos de origen faringoamigdalar. Un título normal de ASLO no descarta el origen postestreptocócico de una 2. 2. glomerulonefritis aguda. Tras una infección estreptocócica, la elevación del título de ASLO aumenta el 3. 3. riesgo de desarrollar una glomerulonefritis aguda. El tratamiento precoz por antibióticos de la infección estreptocócica inicial puede 4. 4. decapitar la esperada elevación del título de ASLO. Gráfico de respuestas 1.

1.

Comentario ! !

! Pregunta de elevada dificultad acerca de un aspecto concreto de la GN aguda post-estreptocócica: el ASLO o antiestreptolisina O. En referencia al diagnóstico de la GNA, llama la atención que los cultivos de faringe o de piel sólo resultan positivos para el estreptococo betahemolítico en alrededor de una cuarta parte de los casos no tratados previamente con antibióticos. Por el contrario, las pruebas serológicas de una infección estreptocócica reciente suelen ser positivas en más del 75% de las GNA. En la mayoría de los casos de origen faringoamigdalar se observa una elevación significativa del título de antiestreptolisina O (ASLO) al cabo de 3-5 semanas del comienzo de la infección. Dicha elevación no guarda relación con el riesgo de desarrollar glomerulonefritis ni con su gravedad o pronóstico (LA OPCION FALSA ES LA 3). En los casos de infección cutánea, el título de ASLO se eleva poco, siendo mucho más frecuente el hallazgo de elevaciones significativas de los títulos de antihialuronidasa, de antiestreptocinasa y de antiDNAasa B.(R/3)

320. Mujer de 20 años que acude a su consulta refiriendo menstruaciones irregulares que incluyen baches amenorreicos. No relaciones sexuales. Antecedentes personales: enfermedades propias de la infancia; amigdalectomía; no alergias medicamentosas conocidas. Exploración: en el tacto rectal parece palparse un aparato genital normal; resto de exploración física sin datos de interés salvo talla de 162 cm y peso de 75 kg. Se realiza ultrasonido que muestra un útero regular de 60 mm y unos anexos ligeramente aumentados de tamaño con múltiples imágenes econegativas compatibles con folículos, la mayor de los cuáles tiene 9 mm de diámetro. Señale la respuesta INCORRECTA: Lo más probable es que encontremos en los exámenes de laboratorio un cociente LH/FSH mayor de 2. El tratamiento de elección de esta paciente se basa en la administración de 2. 2. antiandrógenos tipo acetato de ciproterona. 3. Esta paciente mejoraría su cuadro clínico con una simple reducción del IMC. 3. 4. Algunas pacientes pueden presentar Acantosis nigricans. 4. Gráfico de respuestas 1.

1.

Comentario Sospeche un SOP cuando, como en este caso, se encuentre con una mujer con menstruaciones irregulares y baches amenorreicos, sobrepeso, y hallazgos ecográficos de quistes en ovarios. Los antiandrógenos no son el tratamiento de elección en estas pacientes. Debe iniciarse el tratamiento con reducción de peso y metformina, y anticonceptivos orales si no desea gestación. El acetato de ciproterona, un progestágeno con efecto antiandrógeno, está indicado para el tratamiento del hirsutismo y el acné en estas pacientes.(R/2)

321. Señale cuál de las siguientes respuestas acerca de la problemática social del niño NO es correcta: Cuando se adopta a un niño debe procurarse que sea en la edad más cercana al nacimiento para favorecer el vínculo entre padres e hijo. Se debe informar al niño de su situación en cuanto pueda comprenderlo, como 2. 2. muy tarde a los 4 años. Los niños pequeños no deben asistir a las ceremonias que rodean a la muerte de 3. 3. un progenitor porque crean estado de confusión al no comprenderlo. La reacción más frecuente de los adolescentes frente a la separación o divorcio de 4. 4. los padres, es el pensamiento mágico de que sus padres van a volver a unirse. Gráfico de respuestas 1.

1.

Comentario ! !

! Esta pregunta acerca de la problemática social del niño carece de relevancia. Ante la separación o divorcio de los padres el determinante mas importante de las respuesta iniciales es la edad del niño. Algunos niños presentan depresión, otros lo niegan y evitan el tema como principal mecanismo de defensa. La mayoría se aferra a la fantasía de que la separación no es real y de que volverán a unirse. En relación a la muerte de un ser querido simplemente recuerde que la edad a la que los niños comprenden que la muerte es permanente y comienzan a entenderla como un proceso biológico es entre los 5 y 10 años. El fallecimiento de un familiar es la pérdida más dura para un niño y la reacción ante ella está también esta en función de la edad. Está demostrado que los niños necesitan hablar de la pérdida con adultos, pues les alivia las penas. El que los niños más pequeños no comprendan todas las dimensiones de la muerte no es razón para que se les oculte y no asistan a las ceremonias que la rodean (R/3)

322. Mujer de 78 años de edad, con antecedentes de HTA en tratamiento con lisinopril, que ingresa en unidad coronaria por IAM inferior evolucionado no revascularizado. La paciente rechaza hacerse coronariografía, puesto que se encuentra asintomática y le da miedo la prueba. A las 48 horas de ingreso, la paciente comienza con disnea brusca, intolerancia al decúbito, saturación O2 85% a pesar de O2 en mascarilla y le cae la TA a 85/60 mmHg. En la exploración física, la paciente está sudorosa, mal perfundida y con crepitantes hasta campos medios. El puso es débil, rítmico a 95 lpm, y se le ausculta un soplo pansistólico rudo intenso en borde esternal izquierdo irradiado en barra. Ante la sospecha de complicación mecánica, ¿qué NO esperaría encontrar en esta paciente?: 1. Salto oximétrico en VD. 2. Ondas V grandes en el registro de PCP con el catéter de Swan-Ganz. 3. Acinesia inferior con hipercontractilidad del resto de segmentos no infartados. 4. Índice cardíaco < 2.2 l/min/kg. Gráfico de respuestas

1. 2. 3. 4.

Comentario Generalmente el soplo de IM aguda del caso anterior son soplos que se oyen mal. Sin embargo, el soplo de la rotura del SIV (séptum interventricular) es un soplo como el descrito en el enunciado. Por tanto, la sospecha es de rotura del SIV, precisamente la complicación que es “más rara”. Lo que no veremos son las ondas V grandes en el registro de PCP porque esto es lo que se ve en el caso de que hubiera sido una IM aguda severa (la sangre retrocede del VI a la AI y de aquí a través de las venas pulmonares al capilar pulmonar, produciendo unas ondas V gigantescas). Todo lo demás se puede ver en la rotura del SIV.(R/2)

323. Un paciente es intervenido quirúrgicamente de forma urgente por sospecha de apendicitis aguda. Durante la cirugía se aprecia apendicitis aguda con peritonitis fecal y una formación purulenta localizada. ¿Qué actitud debe adoptarse con los antibióticos utilizados profilácticamente?: 1. Suspender tras la cirugía. 2. Mantener 24-48 horas. 3. Poner otra dosis postoperatoria. 4. Mantenerlos como tratamiento durante 7-10 días. Gráfico de respuestas

1. 2. 3. 4.

Comentario Pregunta fácil y deducible que no debe plantarle mucha duda. En el tratamiento de la apendicitis aguda, la cirugía se complementa con la administración de antibioterapia PROFILACTICA, frente a

! !

! gramnegativos y anaerobios (flora saprófita de colon). Cuando la apendicitis progresa a peritonitis el tratamiento antibiótico ya no es profiláctico sino TERÉEUTICO y debe estar dirigido contra los mismos microorganismos, siendo una combinación útil cefalosporina + metronidazol, por ejemplo.(R/4)

324. Among the following cyanotic congenital heart diseases, only one does NOT typically present with decreased pulmonary flow: 1. Tricuspid atresia. 2. Single ventricle without pulmonary stenosis. 3. Pulmonary atresia with intact ventricular septum. 4. Transposition of the great arteries with pulmonary stenosis. Gráfico de respuestas

1. 2. 3. 4.

Comentario Para contestar preguntas de este tipo es muy importante que tengan clara una clasificación de las cardiopatías congénitas que las divida en cianóticas - acianóticas y, a su vez, con flujo pulmonar aumentado o normal. Ésta es una pregunta difícil pero que puede deducir fácilmente teniendo en cuenta en qué consiste la malformación en cada una de las opciones. Tengan en cuenta que, en general, el flujo pulmonar estará disminuido en aquellas cardiopatías en las que exista una atresia o estenosis de válvulas derechas (tricúspide o pulmonar), como ocurre en las opciones 1, 3 y 4. En el ventrículo único sin estenosis pulmonar, en la que existe una única cavidad ventricular que recibe sangre de las dos aurículas, ocurre lo contrario, es decir, existirá plétora pulmonar.(R/2)

325. Respecto al cáncer de cérvix, cuál de las siguientes afirmaciones es INCORRECTA: 1. El síntoma más frecuente es la leucorrea maloliente. 2. La lesión precursora es la displasia o carcinoma in situ. Entre pacientes con cáncer cervical in situ que no han sido tratadas, del 30% al 3. 3. 70% desarrollarán carcinoma invasor en un período de 10 a 12 años. El cáncer cervical progresa de manera ordenada, pero ocasionalmente se puede 4. 4. presentar un tumor pequeño con metástasis a distancia. Gráfico de respuestas 1. 2.

Comentario El cáncer de cérvix en la mayoría de los casos permanece asintomático. En etapas tardías, el síntoma más precoz y característico es la metrorragia. También es posible que aparezca leucorrea, que en la enfermedad avanzada, es fétido, llamado "en agua de lavar carne".(R/1)

326. Mujer en el 8º mes de gestación que de forma brusca e indolora padece una metrorragia de sangre líquida y roja, de moderada cuantía. El cese del sangrado es espontáneo y cuando usted la explora, ni la madre ni el feto se encuentra afectados. Señale la respuesta FALSA: 1.! El!tratamiento!consiste!en!reposo!en!cama!y!beta 10 mm. 4. La reacción debe leerse a las 24 horas. Gráfico de respuestas

1. 2. 3. 4.

Comentario Teng cuidado con esta pregunta, porque es más difícil de lo que parece. La 2 es correcta. Efectivamente, nadie se sensibiliza por haberse sometido a la intradermorreacción de Mantoux. Existen casos en los que, con un primer Mantoux, el resultado es negativo, siendo el segundo positivo, pero se trata de sujetos previamente sensibilizados en los que se ha producido un fenómeno de “olvido” inmunológico. Al realizarse la prueba por segunda vez, ya positiviza, debido a que el primer Mantoux sirvió como “recordatorio” a la inmunidad (efecto Booster).(R/2)

! !

! 418. ¿En cuál de las siguientes manifestaciones clínicas se requiere el acetato de ciproterona como tratamiento de elección?: 1. Defectos luteínicos. 2. Amenaza de aborto. 3. Hirsutismo manifiesto. 4. Endometriosis. Gráfico de respuestas

1. 2. 3. 4.

Comentario Es una pregunta importante en cuanto que debe conocer un síndrome, el del ovario poliquístico y el hirsutismo es una de sus manifestaciones de la que debe conocer el tratamiento. Se realiza con anticonceptivos orales, que disminuyen la producción de esteroides suprarrenales y ováricos, reduciendo el hirsutismo en 2/3 de los pacientes. A veces se añaden antiandrógenos (espironolactona, acetato de ciproterona, flutamida, cimetidina, finasteride...).(R/3)

419. ¿En cuál de las siguientes patologías NO hay participación del cérvix?: 1. Vaginosis. 2. Sífilis. 3. Gonococo. 4. Molluscum contagiosum. Gráfico de respuestas

1. 2. 3. 4.

Comentario Las 3 primeras opciones pueden producir cervicitis tanto de manera especifica como inespecífica. En cambio la infección por Molluscum contagiosum es de genitales externos.(R/4)

420. En un lactante de dos meses con ictericia apreciable desde la 2ª semana de vida, evacuaciones de color blanquecino, hepatomegalia de 6 cm de consistencia dura, esplenomegalia de 3 cm, en el que se realizó una gammagrafía hepática con HIDA observándose una captación normal en el hígado con ausencia de paso a intestino, ¿cuál es la primera actitud a tomar? 1. Intervención quirúrgica de Kasai. 2. Trasplante hepático. 3. Laparotomía exploradora urgente con biopsia hepática y colangiografía. Dieta hipercalórica con triglicéridos de cadena media, vitaminas A, D, E y K, 4. 4. fenobarbital y colestiramina. Gráfico de respuestas 1. 2. 3.

Comentario La presencia de ictericia, acolia y coluria caracteriza un síndrome colestásico. Éste puede deberse a causas intra o extrahepáticas, y para distinguirlo se ha realizado una gammagrafía con HIDA (ácido hidroxi- indol- acético). Dado que existe un retraso en la evacuación del contraste, el problema muy probablemente sea extrahepático, que es lo que impide que llegue al intestino. Sabiendo esto, el siguiente paso sería conocer cuál es la causa exacta, por lo que elegiremos la respuesta 3. Las opciones 1 y 2 no son válidas porque, hasta este momento, sólo sabemos que tiene una colestasis de origen extrahepático, pero no el diagnóstico exacto, por lo que aún no podemos establecer el tratamiento adecuado.(R/3)

! !

! 421. Atiende en el lugar del accidente a un politraumatizado que se encuentra inconsciente, sin ventilación espontánea, presenta TA 80/50, varias heridas sangrantes en extremidades y fractura abierta de tibia derecha. La actitud inmediata es: 1. Despejar la vía aérea e intubar. 2. Canalizar vía central y administrar expansores plasmáticos. 3. Controlar la hemorragia con torniquetes. Colocar collarín cervical y descartar lesión medular con exploración neurológica 4. 4. completa. Gráfico de respuestas 1. 2. 3.

Comentario Esta pregunta es muy importante ya que el manejo del politraumatizado es un tema muy preguntado en el ENARM. Casi siempre se pregunta del mismo modo, le piden la actitud prioritaria ante un accidentado; se responde conociendo el acrónimo ABCDE, y sabiendo q lo primero que hay que hacer ante un politraumatizado es controlar la vía aérea (Airway), con la extracción de cuerpos extraños y elevando la barbilla, y si es necesario intubar. Conociendo que esta es la primera medida se responden bastantes de las preguntas de este tema.(R/1)

422. La frecuencia de la convulsión febril es: 1. 20%. 2. 8%. 3. 20%. 4. 5%. Gráfico de respuestas

1. 2. 3. 4.

Comentario Las crisis febriles aparecen en un 5% de los niños, y son un proceso típico de la edad infantil (entre los 3 meses y los 5 años), que se relaciona más frecuentemente con el aumento de la temperatura, lo que da lugar a una crisis el primer dia del proceso febril, independientemente del origen del mismo. NO SE RELACIONAN CON UN MAYOR RIESGO DE EPILEPSIA.(R/4)

423. ¿Cuál de los siguientes cuadros clínicos NO mejoraría su agudeza visual con gafas?: 1. Miopía. 2. Hipermetropía. 3. Presbicia. 4. Astigmatismo irregular. Gráfico de respuestas

1. 2. 3. 4.

Comentario Pregunta sencilla aplicando conocimientos básicos de defectos de refracción. Las gafas se usan para corregir los defectos de refracción. Usamos lentes esféricas principalmente para la corrección de la miopía, hipermetropía y la presbicia. Las lentes cilíndricas se usan, de forma general, para corregir el astigmatismo asociado o no a otras ametropías. Sin embargo en el astigmatismo irregular, en el que no existen focos claros definidos, no es posible la corrección con lentes convencionales. La mejor corrección posible se consigue con la utilización de lentes de contacto rígidas.(R/4)

424. Las metástasis inguinales en el cáncer epidermoide de ano:

! !

! 1. Son siempre síncronas con el tumor primario. 2. Son susceptibles de disección profiláctica. 3. Son susceptibles de disección terapéutica. 4. No existe diseminación inguinal en el cáncer de ano. Gráfico de respuestas

1. 2. 3. 4.

Comentario Los tumores anales son relativamente raros. Más frecuentes en hombres jóvenes. La mayoría son asintomáticos, siendo el dolor y la hemorragia las principales manifestaciones. El más frecuente es el carcinoma de células escamosas. Se suele presentar como ulceración del borde anal, de hecho toda ulceración crónica en la región anal que no cicatriza debe considerarse un carcinoma hasta que no se demuestre lo contrario con biopsia. Tiene tendencia a la diseminación hacia ganglios inguinales. El tratamiento es la escisión local amplia o la radioterapia, con similares resultados. Se lleva a cabo una linfadenectomía inguinal en caso de que haya ganglios positivos. Es raro que se realice amputación abdominoperitoneal.(R/3)

425. Los signos de hipoxia fetal son indicación de administración de O2 a concentraciones elevadas a la madre, y de inducción del parto. Entre estos hallazgos se encuentran los siguientes, EXCEPTO: 1. Deceleración variable o tardía en la frecuencia cardíaca fetal. Acidosis de tipo metabólico y respiratorio, en el análisis de sangre del cuero 2. 2. cabelludo, con pH menor de 7.20. 3. 3. Bradicardia fetal con disminución de la variabilidad entre cada latido. 4. Disminución de la resistencia vascular fetal detectada con USG doppler. 4. Gráfico de respuestas 1.

Comentario La hipoxia fetal se asocia además a acidosis. Ambas inducen en el feto bradicardización, disminución de la variabilidad, CIR y elevación de las resistencias vasculares fetales (no dismunción de las mismas, como aparecen en la opción 4). Este hecho se puede detectar a través de USG con sonda de doppler.(R/4)

426. ¿Cuál de las siguientes alternativas es una medida eficaz en el tratamiento de la enfermedad diarreica aguda en niños? 1. Difenoxilato y atropina. 2. Loperamida. 3. Rehidratación oral. 4. Racecadotrilo. Gráfico de respuestas

1. 2. 3. 4.

Comentario Por nada del mundo puede fallar el abordaje de diarrea en pediatría. La rehidratación oral es una de las medidas más eficaces en el tratamiento del síndrome diarreico agudo. Recordemos que el suero oral hizo merecedores del Nobel a sus inventores. Respuesta 3 correcta. No es necesario administrar loperamida.

! !

! Composición del vida suero oral es de: glucosa 111mEq/lt, sodio 90mEq/lt, cloruro 80 mEq/lt, citrato 30 mEq/lt, potasio 20 20 mEq/lt, HCO3 30 mEq/lt. Existen 3 planes de rehidratación dependiendo el grado de deshidratación. PLAN A Para prevenir la deshidratación y desnutrición: Se aplica en paciente con diarrea aguda NO deshidratados. - Alimentación Continúa A - Bebidas abundantes B - Consulta educativa C Dar “Vida Suero Oral” VSO de la siguiente de la siguiente manera: Dar 1 taza (150 ml) después de cada evacuación en niños mayores de 1 año y media taza (75 ml) después de cada evacuación en niños menores de 1 año. Se ofrece a cucharaditas o sorbos. Se realiza en domicilio. PLAN B El suero oral se administra a 100ml/Kg de peso en cuatro horas.La dosis se fracciona en tomas cada 30 minutos y se ofrece lentamente con taza y cucharita para no sobrepasar la capacidad gástrica y así disminuir la probabilidad del vómito. La rehidratación con este plan puede durar de 2 a 8 hrs. en el hospital. PLAN C Es el plan de rehidratación intravenosa que se emplea en la deshidratación severa. Se solución Hartmann y si esta no esta disponible se utiliza solución salina al 0.9% en un esquema para tres horas. Primera hora : 50 ml/kg/hora Segunda hora : 25 ml/Kg/hora Tercera hora : 25 ml/Kg/hora. (R/3)

427. Femenino de 44 años que acude a la consulta de ginecología por menorragias, sin otra sintomatología acompañante. La exploración ginecológica muestra vagina y cérvix normales, mamas normales y tacto vagino-abdominal con útero aumentado de volumen, de consistencia dura y superficie irregular. El USG confirma el diagnóstico de tres miomas de entre 3-4 cm de diámetro, de localización submucosa. En los exámenes de

! !

! laboratorio se detecte una anemia ferropénica moderada. ¿Qué tipo de tratamiento propondría a esta paciente? 1. Miomectomía abdominal. 2. Análogos de GnRH. 3. Tratamiento médico de la anemia ferropénica y observación de los miomas. 4. Tratamiento de la anemia ferropénica e histerectomía abdominal. Gráfico de respuestas

1. 2. 3. 4.

Comentario Nos presentan el caso de una paciente con miomas, en la que éstos tienen repercusión clínica: menorragias y anemia ferropénica. Si tratamos la anemia exclusivamente con hierro, lo más probable es que reaparezca pasado un tiempo, porque la causa persistiría. Por ello, hay que ofrecerle una solución para el problema de base, que son los miomas. Los análogos de la GnRH no son el tratamiento definitivo de los miomas. Se utilizan previamente a la cirugía, porque disminuyen su tamaño, pero no de forma aislada. Por otra parte, el tipo de cirugía preferible en esta paciente es la histerectomía, puesto que ya no se encuentra en edad reproductiva. La miomectomía se reserva para pacientes jóvenes que todavía no han cumplido sus deseos genésicos.(R/4)

428. Los requerimientos diarios de insulina en los pacientes con DM tipo 1 disminuyen: Entre uno y seis meses después del diagnóstico, bajo tratamiento con insulinoterapia intensificada. 2. 2. Ante una apendicectomía. 3. 3. Durante la adolescencia. 4. Por la presentación asociada de hipertiroidismo. 4. Gráfico de respuestas 1.

1.

Comentario Tras el debut de una diabetes mellitus tipo 1 puede existir un periodo de unos meses de duración, conocido como "luna de miel" en el que todavía se mantiene cierta reserva insulínica propia y se puede reducir o incluso suspender el aporte de insulina. En la adolescencia, por el efecto de la GH como hormona contrarreguladora, puede ser necesario tener que aumentar las dosis de insulina (fenómeno del alba). En el resto de las respuestas, también va ser necesario tener que aumentar la insulina porque corresponden a situaciones de estrés.(R/1)

429. La hemorragia uterina disfuncional puede ser controlada administrando la hormona faltante que generalmente es: 1. Estrógenos. 2. Testosterona. 3. Progesterona. 4. Tiroideas. Gráfico de respuestas

1. 2. 3. 4.

Comentario En el tratamiento de la hemorragia uterina disfuncional se controla con la administración de progesterona únicamente o combinada, por lo que la respuesta correcta es la 3.

! !

! 430. ¿Cuál de los siguientes tumores produce virilización?: 1. Tecoma. 2. Gonadoblastoma. 3. Tumor de Brenner. 4. Tumor de seno endodérmico. Gráfico de respuestas

1. 2. 3. 4.

Comentario Esta pregunta de tumores de ovario es muy sencilla y se responde con conocimientos básicos como que los andrógenos son los responsables de la virilización y en el ovario se producen en la teca, por lo que un tumor de esta capa producirá virilización. Por el contrario, un tumor de la granulosa produce estrógenos y producirá feminización. El gonadoblastoma, el disgerminoma y el tumor del seno endodérmico, procedentes de la célula germinal y el tumor de Brenner, con epitelio transicional de vejiga no secretan hormonas esteroideas. Debe recordar que la causa de virilización de origen ovárica más frecuente es el androblastoma.(R/1)

431. Respecto a las diferencias entre la leche materna y la leche de vaca, señale la afirmación CORRECTA: 1. La leche de vaca contiene una cantidad de calcio superior a la leche materna. 2. La leche materna contiene mayor cantidad de vitamina K que la leche de vaca. 3. La leche de vaca es mas rica en hidratos de carbono que la leche materna. 4. La leche materna tiene una relación caseína/seroproteinas de 70/30. Gráfico de respuestas

1. 2. 3. 4.

Comentario Aunque es una pregunta que relativamente carece de importancia, es un dato curioso saber que la leche de vaca contiene más calcio que la leche materna.(R/1)

432. Paciente de 22 años que acude por primera vez a la consulta para realizar una revisión ginecológica. Menarquia a los 14 años,

! !

! reglas normales, no dolorosas, nuligesta. Se realiza USG vaginal, objetivando en región anexial izquierda lo que se muestra en la imagen. Señale lo INCORRECTO: 1. Es el tumor germinal ovárico más frecuente. 2. Pueden producir alfafetoproteína. 3. Son benignos. 4. Habitualmente cursan con dismenorrea. Gráfico de respuestas

1. 2. 3. 4.

Comentario Teniendo en cuenta que se trata de una revisión rutinaria, en una mujer joven, sin factores de riesgo para tumoración maligna y, por lo demás, asintomática, aparte de las características de la imagen, debemos concluir que seguramente se trata de un quiste ovárico totalmente benigno. Observe que se trata de una tumoración completamente regular, tanto en su contorno (redondeado y liso) como en su contenido. Estos quistes suelen ser, simplemente, hallazgos incidentales y casi siempre son asintomáticos, por lo que la respuesta incorrecta es la 4.

433. Usted decide intervenir a la paciente del caso clínico anterior con fines diagnósticoterapéuticos. Señale la opción CORRECTA: Los análogos de GnRH tras la intervención pueden resultar eficaces para el control de la enfermedad. La anexectomía laparoscópica es el tratamiento más idóneo, debido al contenido 2. 2. de la tumoración. La quistectomía laparoscópica sin biopsia intraoperatoria es el tratamiento más 3. 3. oportuno. 4. 4. En los embarazos suelen disminuir de tamaño. Gráfico de respuestas 1.

1.

Comentario Difícilmente vamos a poder tratar un quiste (tumoración benigna, pero tumoración) con tratamiento médico. Por ello, lo que debemos hacer es extirparlo, lo cual hoy día es posible mediante cirugía laparoscópica. Dada la juventud de la paciente y las características ecográficas del quiste, no precisamos biopsia intraoperatoria, ya que se trata de una neoplasia benigna casi con total seguridad. Recordemos que una tumoración que sí puede reducir su tamaño con tratamiento médico son los miomas, utilizando agonistas de la GnRH. No obstante, el mioma no tendría un contenido líquido (anecoico, como en este caso), sino sólido. Y, en cualquier caso, lo que se suele conseguir es que reduzcan su tamaño, pero para que desaparezcan por completo sería necesaria la extirpación quirúrgica.(R/3)

434. Durante el embarazo, la artritis reumatoide suele: 1. Afectar las rodillas. 2. Mejorar. 3. Producir manifestaciones renales. 4. Producir nódulos. Gráfico de respuestas

1. 2. 3. 4.

Comentario ! !

! Aproximademente el 70-80 % de las pacientes con AR experimenta una remisión clínica o una mejoría sustancial durante el embarazo. A pesar de ello la paciente requiere alguna intervención terapeútica la medida de elección son los corticoides que no sea de larga vida media, ya que no atraviesan las barrera placentaria. Los AINEs se podrían utilizar si el beneficio superase al riesgo pero habitualmente se pueden sustituir por corticoides a dosis bajas. Los AINEs pueden provocar el cierre precoz del conduto arterioso, prolongar la gestación y el parto e inducir hemorragias intracraneales en prematuros. Dentro de los fármacos modificadores de la enfermedad se deben evitar como normal general. El más seguro sería no obstante la sulfasalazina.(R/2)

435. En relación a los divertículos colónicos, señala el enunciado FALSO: 1. Su incidencia aumenta con la edad. 2. Son más frecuentes a nivel de sigma. 3. Son protusiones en fondo de saco de toda la pared intestinal. 4. La hemorragia ocurre sólo en un pequeño porcentaje de pacientes. Gráfico de respuestas

1. 2. 3. 4.

Comentario Los divertículos de colon son herniaciones de la mucosa colónica a través de las capas musculares de la pared del colon. La enfermedad diverticular, o diverticulosis, es más frecuente en personas ancianas. Se localizan fundamentalmente en colon descendente. Suelen ser asintomáticos. Pero cuando dan clínica, lo más frecuente es la hemorragia (que generalmente es de divertículos de colon ascendente).(R/3)

436. Mujer de 35 años que acude a su revisión ginecológica rutinaria, con ciclos regulares, siendo su última regla hace 17 días. En el ultrasonido transvaginal se observa un útero en retroflexión, regular, con endometrio lineal de segunda fase y en uno de los ovarios se observa una imagen econegativa de 31 mm, sin otros hallazgos de interés. Ante esta situación usted decide: 1. Completar el estudio con marcadores tumorales. Completar el estudio con pruebas radiológicas más específicas como resonancia 2. 2. magnética o TAC pélvico. 3. Nuevo ultrasonido de control en 4-6 semanas. 3. 4. Cirugía citorreductora. 4. Gráfico de respuestas 1.

Comentario La imagen que nos describen en el caso clínico hace referencia al diagnóstico casual de un quiste de contenido líquido, puesto que es econegativo, en el contexto de una mujer en edad fértil. Lo más probable que dicha imagen corresponda con un folículo funcional, por el periodo del ciclo en que se encuentra la paciente, por lo que lo más correcto sería repetir la exploración en 1 ó 2 meses para evidenciar como dicha imagen ha desaparecido.(R/3)

437. El tumor uterino que tiene mayor riesgo de hemorragia es el: 1. 2.

! !

1. Subseroso. 2. Intramural.

! 3. Intraligamentario. 4. Submucoso. Gráfico de respuestas

3. 4.

Comentario Los miomas submucosos se asocian con un aumento del sangrado menstrual y son más sintomáticos que los demás. Respuesta 4 correcta. Miomas subserosos (40%): situados bajo el peritoneo visceral uterino . Pueden alcanzar gran tamaño, siendo por su localización poco sintomáticos. Miomas intramurales: son los más frecuentes (55%) y proliferan enla porción central del miometrio. Miomas submucosos (5-10%): hacen protrusión en la cavidad uterrina, por lo que son los más sintomáticos. Pueden ser pediculados y prolapsarse a través del orificio cervical (se habla entonces de mioma parido)(R/4)

438. Los gemelos unidos se forman cuando: Entre el 4º y 8º día postfecundación antes de la diferenciación de las células del amnios. 2. La división ocurre después de la formación del disco embrionario. 2. Si la división ocurre cuando el amnios ya está establecido, 8 días después de la 3. 3. fertilización. 4. 4. Los gemelos son dicigóticos, pero existe una anomalía en el desarrollo amniótico. Gráfico de respuestas 1.

1.

Comentario Existen dos tipos de embarazo gemelar: dicigótico (70%) y monocigótico (30%). Los gemelos monocigóticos son genéticamente idénticos. Entre los monocigóticos, algunos nacen unidos en grados muy diversos, desde una pequeña conexión superficial hasta compartiendo órganos vitales. La división de los embriones de los gemelos independientes se produce durante los diez días siguientes a la fecundación. En cambio, en los gemelos unidos se produce más tardíamente, cuando el disco embrionario ya está formado. Por eso, la división será incompleta, y al nacimiento se manifiesta como una conexión anatómica entre los dos gemelos.(R/2)

439. La lubricación de la vagina durante el coito, se debe a: 1. Trasudado vaginal. 2. Secreción de glándulas de Bartholino. 3. Flujo cervical. 4. Fluido endometrial. Gráfico de respuestas

1. 2. 3. 4.

Comentario La lubricación vaginal se produce por la segregación del flujo vaginal, un líquido viscoso segregado por el cuello de la matriz y las paredes de la vagina con diversos fines. La lubricación de los labios, en cambio, se debe a las glándulas de Bartolino. Respuesta 1 correcta.

! !

! 440. Señale el cuadro clínico que NO esperaría encontrar en un adolescente con fibrosis quística: 1. Diabetes mellitus. 2. Azoospermia obstructiva. 3. Amenorrea primaria. 4. Pólipos nasales. Gráfico de respuestas

1. 2. 3. 4.

Comentario Es importante conocer los aspectos clínicos mas relevantes de la fibrosis quística. La alteración de la proteína reguladora de la conductancia transmebrana origina excreción defectuosa de cloro y excesiva reabsorción de sodio, lo que causa secreciones muy espesas en diferentes órganos y la subsiguiente obstrucción de sus conductos (páncreas, epidídimo, hígado, vía biliar, intestino, tracto respiratorio...). El movimiento de los cilios alterado por la alta viscosidad de las secreciones facilita la colonización de gérmenes patógenos en el epitelio respiratorio, lo que ocasiona una respuesta inflamatoria crónica que puede dar lugar a pólipos nasales. A menudo existe amenorrea secundaria a consecuencia de la enfermedad crónica y la reducción marcada del peso corporal, pero no produce amenorrea primaria.(R/3)

441. Un hombre de 36 años de edad es valorado porque tiene una pierna roja, caliente y dolorosa, desde hace cuatro días, con dolor e impotencia funcional. Sin antecedentes familiares ni personales de interés, ni tampoco traumáticos ni quirúrgicos sobre dicha pierna. La eco-Doppler de dicha pierna nos muestra un trombo en la vena poplítea. Se le trata con heparina de bajo peso molecular y, posteriormente, comienza con anticoagulantes orales. Un estudio posterior revela que este paciente es portador de la mutación asociada al factor V de Leyden. ¿Cuál de las siguientes pautas de duración del tratamiento anticoagulante es la más adecuada para este paciente?: 1. 2 meses. 2. 1 mes. 3. Toda la vida. 4. 2 años. Gráfico de respuestas

1. 2. 3. 4.

Comentario Los pacientes con una mutación en el factor V de Leyden que sufren un episodio tromboembólico tienen mucho riesgo de recurrencia, por lo que la anticoagulación debe mantenerse de por vida, al no tratarse de una condición reversible.(R/3)

442. ¿Cuál de las siguientes NO se considera un indicador de alto riesgo en el TEP?: 1. Hipotensión mantenida. 2. Choque cardiogénico. 3. Insuficiencia respiratoria. 4. Elevación de troponina. Gráfico de respuestas

1. 2. 3. 4.

Comentario La valoración del riesgo en pacientes con TEP es un elemento básico para decidir el tratamiento. El tratamiento primario (fibrinólisis) se reserva para pacientes de alto riesgo (hipotensión o choque

! !

! cardiogénico) y en pacientes con riesgo intermedio (disfunción de VD o elevación de marcadores de daño miocárdico) se debe valorar de forma individualizada la relación riesgo/beneficio de su empleo. Los pacientes con bajo riesgo se benefician de tratamiento anticoagulante. La insuficiencia respiratoria no se contempla como indicador de alto riesgo.(R/3)

443. Dentro del tratamiento no farmacológico de la HTA lábil se incluyen las siguientes medidas dietéticas, EXCEPTO: 1. Dieta sin sal. 2. Reducción de peso. 3. Ejercicio físico isométrico. 4. Suspender la toma de anticonceptivos orales. Gráfico de respuestas

1. 2. 3. 4.

Comentario Son medidas no farmacológicas que disminuyen la PA: - Restricción de la ingesta de sal - Disminución de peso - Ejercicio físico aeróbico - Evitar el estrés La suspensión de fármacos causantes de HTA, como los anticonceptivos orales, también ayuda a controlar la PA. La supresión del tabaco no reduce los niveles de PA, pero reduce el impacto de la HTA sobre los órganos diana. El ejercicio isométrico (levantar pesas) no se recomienda como método para tratar la HTA, pues mal realizado puede incluso incrementarla (al realizar Valsalva).(R/3)

444. Paciente femenino de 30 años consulta por un nódulo mamario que se diagnostica de carcinoma ductal infiltrante. Tiene antecedentes familiares, ya que su madre y su abuela han padecido esta misma enfermedad. El estudio genético muestra la presencia de BRCA1, c-erb B2 y alteraciones de la p53. Receptores estrogénicos positivos. Sobre las características citadas de esta paciente, señale cuál puede considerarse un factor de buen pronóstico: 1. Presencia de receptores estrogénicos positivos. 2. Alteraciones en p53. 3. Edad inferior a 35 años. 4. Presencia de BRCA1. Gráfico de respuestas

1. 2. 3. 4.

Comentario Tanto las alteraciones en el p53, antecedentes familiares, la positividad para c-erb B2, la edad menor de 35 años y la presencia de BRCA1 son factores demostrados de mal pronóstico. Por el

! !

! contrario, es la AUSENCIA de receptores hormonales la que confiere un mal pronóstico y no la presencia de los mismos, como indica la opción 1.(R/1)

445. Paciente femenino de 29 años, con historia de alteraciones del ciclo frecuentes, consistentes en oligomenorrea, hirsutismo en la escala de Ferriman-Galley de 11 puntos, IMC 31 y lesiones verrugosas aterciopeladas y pigmentadas en nuca, axila y pliegue submamario. Respecto a la patología que sospecha, sólo con la clínica, es FALSO que: 1. Muy probablemente padezca resistencia insulínica. 2. El cociente LH/FSH es superior a 2. 3. Tiene riesgo incrementado de padecer patología endometrial. 4. La primera medida a tomar será la pérdida de peso. Gráfico de respuestas

1. 2. 3. 4.

Comentario Se presenta un caso clínico típico de una paciente con síndrome de ovario poliquístico (SOP), la cual presenta alteraciones menstruales por defecto e hirsutismo. Se considera hirsutismo en la escala de Ferriman- Galley una cifra superior a 8. Además presenta en piel lesiones de acantosis nigricans, las cuales implican resistencia insulínica en el 90% de las veces. Además hay un hiperandrogenismo muy importante. En estas pacientes obesas con resistencia insulínica, el cociente LH/FSH suele estar próximo a 1 o incluso ser superior la FSH. Cuando son pacientes delgadas sin resistencia insulínica, suele ser superior a 2. Estas pacientes, al no ovular y no compensar los estrógenos con progesterona, tienen más riesgo de hiperplasia de endometrio. Obviamente, la primera medida de salud será la pérdida de peso.(R/2)

446. ¿Cuándo se aconseja intervenir quirúrgicamente la hernia umbilical infantil?: 1. Se debe intervenir cuanto antes para evitar el riesgo de estrangulación. A partir de los 3 ó 4 años de vida, al ser frecuente su cierre espontáneo antes de 2. 2. esa edad. Con el fin de utilizar prótesis en la cirugía, se recomienda esperar a que el 3. 3. paciente sea mayor. 4. Si el niño presentara vómitos durante los accesos febriles. 4. Gráfico de respuestas 1.

Comentario Dentro de las masas umbilicales en un recién nacido, la hernia umbilical se clasifica como una masa grande que se puede detectar a nivel del cordón (respetándolo) ya en el periodo neonatal. Está cubierta de piel, es blanda y reductible, aumentando de tamaño en valsalva (cuando el niño llora). Suele desaparecer a los tres años y no se trata antes de esta edad a no ser que esté incarcerada o estrangulada. El onfalocele también es una masa grande pero está cubierta sólo por peritoneo y, a diferencia de la hernia, su tratamiento sí que es quirúrgico.(R/2)

447. Todas las siguientes son posibles manifestaciones de inicio de la fibrosis quística y, por tanto, indicaciones para la realización del test del sudor, EXCEPTO: 1.! ! 2.! !

! !

1.! Tos!crónica.! 2.! Sabor!salado!al!besarle.!

! 3.! Esteatorrea.! 4.! Invaginación!intestinal.! Gráfico de respuestas

3.! ! 4.! !

Comentario La fibrosis quística (FQ) es un tema fundamental para el examen de la que debe conocer prácticamente todo. No obstante, esta es una pregunta muy difícil. La FQ es la enfermedad genética más letal en raza caucásica. Se debe a mutación del gen CFTR (proteína reguladora de la conductancia transmembrana), que está en el brazo largo cromosoma 7. La mutación más prevalente en nuestro medio es la delta- F508. Es una enfermedad con herencia AR, por lo que es necesario tener mutados los dos alelos para expresar la enfermedad. La alteración de la regulación de los canales iónicos de las membranas celulares produce secreciones deshidratadas y espesas, lo que origina la clínica. La afectación pulmonar determina la mortalidad de esta enfermedad. Para el diagnóstico es necesario una sospecha y una confirmación. La sospecha es por la clínica, antecedentes familiares o cribado neonatal positivo (elevación de tripsinógeno inmunoreactivo). La confirmación es mediante el test del sudor, alteración en la diferencia de potencial nasal y estudio genético (este último indicado en menores de 3 meses). En el manual viene una lista con las distintas manifestaciones clínicas que son indicación para realizar el test del sudor. De las 4 respuestas, todas son correctas menos la invaginación intestinal. No la confunda con el íleo meconial, que sí sería sugestivo.(R/4)

448. Mujer de 23 años de edad con diagnóstico de lupus eritematoso sistémico que presenta cifras de hemoglobina de 9.8 g/dl, microcitosis, hierro sérico disminuido, niveles de transferrina reducidos y ferritina sérica elevada. ¿Qué tipo de anemia le parece más probable que presente esta paciente?: 1. Anemia aplásica. 2. Anemia megaloblástica. 3. Anemia de inflamación crónica. 4. Anemia ferropénica. Gráfico de respuestas

1. 2. 3. 4.

Comentario Esta es una pregunta que no debe de fallar, solo con conceptos básicos se puede contestar. Las opciones 2 es una anemia macrocítica, por lo que descartamos con facilidad esta opción. La duda puede surgir entre las opciones 3 y 4, porque ambas pueden cursar con microcitosis; para diferenciarlas siempre debe de fijarse en los valores de ferritina sérica, que expresan como se encuentran los depósitos de hierro en el organismo; en este paciente están elevados, y además nos hablan de una enfermedad crónica como el LES, por lo que la opción 3 es la más correcta.

449. Una mujer de 55 años, posmenopáusica, consulta por astenia y disnea de esfuerzo. En la anamnesis refería ligera epigastralgia y pirosis ocasional. No metrorragias. Laboratorio: Hb 6 g/dl, VCM 69 fl, sideremia 13 microgramos/dl, ferritina 4 ng/ml.

! !

! Endoscopia digestiva alta: pequeña hernia hiatal por deslizamiento sin signos de esofagitis. ¿Cuál es la actitud más CORRECTA con esta enferma? 1. Administrar hierro oral y ver evolución de la anemia. 2. Tratar con inhibidores de la bomba de protones y evaluar a los tres meses. 3. Recomendar una colonoscopia completa. 4. Solicitar un evaluación ginecológica. Gráfico de respuestas

1. 2. 3. 4.

Comentario Se nos presenta paciente de 55 años con anemia ferropénica. Se realiza estudio endoscópico alto en el que se visualiza hernia de hiato sin otros hallazgos lo que no justifica la anemia de la paciente, motivo por el que está indicado un estudio con colonoscopia completa. Con la edad de la paciente y la gravedad de la anemia no está indicado administrar hierro, y esperar, ya que el cuadro es sugestivo de patología orgánica. Por prevalencia es más habitual encontrar lesiones con potencial hemorrágico en colon que en ID por lo que se debe realizar la colonoscopia antes que la cápsula endoscópica.(R/3)

! !

!

! !

!

! !

!

! !

!

! !

!

! !

!

! !

!

! !

!

! !

!

! !

!

!

! !